Download as docx, pdf, or txt
Download as docx, pdf, or txt
You are on page 1of 142

SOAL Masuk 2018

1. Polypeptida hormone which helps gonadotrophin actions in theca and granulosa cells is :
A. Insuline-like growth factors
B. Anti-mullerian hormone
C. Ephinephrine
D. Glucagon
E. Peptin
2. FSH function in follicular phase is :
A. Avoid oocyte maturation inhibitor effect on oocyte
B. Avoid domination of developing follicle
C. Avoid premature lutenization of oocyte
D. Avoid multiple oocyte maturation
E. Avoid apoptosis of follicle
3. Estrogen produced by dominant follicle has an effect :
A. Stimulates negative feedback on LH
B. Inhibits positive feedback on LH
C. Inhibits testerone aromatization on granulosa cell
D. Stimulate FSH receptor formation in dominant follicle
E. Inhibit LH receptor formation in dominant follicle
4. The basic philosophy of pulsatile gonadotrophin secretion is :
A. To avoid downregulation
B. To avoid positive feedback effect
C. To avoid negative feedback effect
D. To avoid accumulation hormone in target tissue
E. To avoid degradation of hormone in bloodstream
5. Estrogen to estradiol conversion is facilitated by :
A. 17beta hydroxysteroid dehidrogenase
B. 3beta hydroxysteroid dehidrogenase
C. P450 side chain cleavage
D. P450 aromatase
E. 5 alfa reductase
6. LH surge incite following events :
A. Meiosis, theca cell luteinization, theca cell expansion, OMI synthesis
B. Meiosis, theca cell luteinization, cumulus expansion, OMI inhibition
C. Meiosis, granulosa cell luteinization, cumulus expansion, prostaglandin synthesis
D. Meiosis, granulosa cell luteinization, theca cell expansion, luteinization inhibitor
synthesis
E. Meiosis, granulosa cell luteinization, granulosa cell expansion, prostaglandin
inhibitor
7. The rise of FSH in preovulatory period has consequence :
A. Release oocyte from its attachment in follicle, convert plasminogen to plasmin, LH
receptor formation
B. Bind oocyte to its attachment in follicle, convert plasminogen to plasmin, FSH
receptor
formation
C. Bind oocyte to its attachment in follicle, convert plasminogen to prostaglandin,
FSH receptor
formation
D. Release oocyte from its attachment in follicle, convert plasminogen to
prostaglandin, LH
receptor formation
E. Release oocyte from its attachment in follicle, convert plasminogen to plasmin,
prostaglandinreceptor formation
8. Acromosome reaction pproduces :
A. Release of enzymes such as hyaluronidase and acrosin
B. Separation between head and tail of spermatozoa
C. Expansion of cumullus cells
D. Separation between midplace and tail of spermatozoa
E. Binding of spermatozoa with zona pellucida
9. Zone reaction will produce ;
A. Zona pellucida become impermeable after fertilization of one spermatozoa
B. Zona pellucida become permeable to allow easy penetration of spermatozoa
C. Zona pellucida axpand to allow easy penetration of spermatozoa
D. Zona pellucida become thin to allow easy penetration of spermatozoa
E. Zona pellucida become thin to produce dense layer of cumullus cells
10. Capacitation is characteristized by :
A. Acrosomal reaction, binding capabilities with zona pellusida, hipermotility
B. Mitochondrial reaction, binding capabilities with cumullus, hipomotility
C. Mitochondrial reaction, binding capabilities with cumullus with zona pellucida,
hipomortility
D. Acrosomal reaction, binding capabilities with cumullus, hipermotility
E. Acrosomal reaction, binding capabilities with zona pellusida, hipomotility
11. The role of progesterone in pregnancy is :
A. Substrate for glucocorticoid and mineralocorticoid production in fetal adrenal
gland
B. Fetal cholesterol pool
C. Substrate for estrogen production in placental compartment
D. Substrate for androgen production in maternal compartment
E. Maternal cholesterol pool
12. Rapid conjugation of sex steroid hormone with sulfate compound is :
A. To convert to more active form of sex steroid hormone
B. To protect the fetus from adverse effect of abudant sex steroid hormone
C. To ensure the plasental sulfatase enzyme in active condition
D. To change to longer action from of steroid hormone
E. To protect the maternal compartment from sex steroid hormone deficiency
13. Cortisol from maternal compartment cannot influence the fetal development due to :
A. Its conversion to progesterone
B. Its conversion to aldosterone
C. Its conversion to cortisone
D. Its conversion to estron
E. Its conversion to DHAS
14. The fetus is protected from masculinization due to high level of androgen during
pregnancy by :
A. Aromatization maternal compartment and conjugation with sulfate compound
B. Aromatization in placental compartment and conjugation with sulfate compound
C. Aromatization in placental compartment and conjugation with hydroxy compound
D. Aromatization in placental compartment and conjugation with glucuronyl
compound
E. Aromatization in fetal ovary compartment and conjugation with hydroxy
compound
15. The decrease of estrogen and progesterone level in late luteal phase lead to:
A. Increase of inhibin a secretion
B. Decrease of pulsatile frequency secretion of GNRH
C. Increase of pulsatile frequency secretion of GNRH
D. Sustained feedback inhibition to hypothalamus until menstruasi phase
E. Sustained feedback inhibition to hypothalamus until follicular phase
16. Progesterone production in pregnancy is dependent from
A. Presence of fetus
B. Presence of placental
C. Presence of fetal ovary
D. Presence of maternal ovary
E. Presence of fetal DHAS production
17. Low estriol level in pregnancy correlation with
A. Fetal macrosomia
B. P45)c17 deficiency
C. Adrenal hypofunction
D. Adrenal hyperfunction
E. 5-OH-steroid dehydrogenase deficiency
18. Wich of the following is the best explaination for breast development in a patient with
androgen
insensitivity?
A. Gonadal production of estrogen
B. Adrenal production of estrogen
C. Breast tissue sensitivity to progesterone
D. Peripheral convertion of androgens
E. Autonomous production of breast-spesific estrogen
19. Miss mawar, 18 year old nulliparous complain of not having started her menses, her
breast
development is Tunner stage IV. She had blind vaginal pouch an no cervix. Which of the
following is
most likely diagnosis?
A. Mullerian agenesis
B. Androgen insensitivity
C. Both of the above
D. Neither of the above
E. Gonadal dysgenesis
20. Miss Beauty, 20 years old with 46, XY karyotype is noted to be a sexuality infantile
phenotypic female and diagnosed as having gonadal dysgenesis. Which the following is the
most important treatment for this patient?
A. Progesteron therapy to reduced osteoporosis
B. Estrogen and androgen therapy to enchance height
C. Progesterone therapy to prevent endometrial cancer
D. Gonadectomy
E. Estrogen therapy to initiate breast development
21. Cytologic hormonal evaluation are sometime expressed by mean of indexes. Which the
following
index is not match for evaluating hormonal cytology?
A. Crowded cell index
B. Eosinophyllic index
C. Maturation index
D. Karyopyknotic index
E. Mitotic index
22. Immunohystochemistry method can detect the specific antigen expresed by certain cells.
Ussualy we
chose the best of the following immunohystochemyitry method
A. Direct method
B. Peroxydase anti peroxydase method
C. Avidin biotin complex method
D. Toluidin method
E. Massion trichrome method
23. Laparascopic ovarian surgery
A. Should be limited to women with PCOS who have and clomiphene intolerance or
resistence
B. Result in live in 60-80 % of women
C. Has similiar multiple pregnancy rates and miscarriage rate to ovalation induction
with FSH
D. Is associated with reduced sensitivity to ovulation induction with FSH
E. All of the above
24. The mecanism of action of induction of ovulation following laparascopic ovarian drilling
is
A. Reduce serum FSH levels
B. No change in inhibin levels
C. Reduction in LH and androgen levels
D. Reduction in circulating insulin and 1GFBP-1 levels
E. All of the above
25. Which of the following statement is true about adenomyosis
A. Adenomyotic tissue contains higher steroid receptor levels than normal
myometrium
B. Adenomyotic often associated with endrometriosis
C. The majority of adenomyosis cases are associated with adenomatous hyperplasia
of the
endometrium
D. Hereditary occurance of adenomyosis is relative frequence
E. Oral contraceptive pills are affective in reducing the uterine volume in
adenomyosis

26. All of the following are possible etiologies of primary amenorrhea, except :
A. Vaginal agenesis
B. Asherman’s syndrome
C. Mayer-rockitansky-kuster-hauser syndrome
D. Testicular feminization
E. Hypogonadotropic hypogonadism
27. Biomolecular technique which allows one small portion of DNA to be replicated and
analyzed is
A. Southern biot analysis
B. Hybridization
C. Polymerase chain reaction
D. Cloning DNA
E. Northern biotting
28. Most of the MHC antigens expression by trophoblast are
A. Erithrocyte antigenic systems
B. HLA-G
C. uNK cells
D. Placenthal alkaline phosphatase
E. Rhesus-O
29. Endometrial receptivity, except :
A. Has an implantation window of 4 days
B. Is worse in stimulated IVF cycles
C. Requires sufficient amounts of chorionic gonadotrophine
D. Depends on matching blastocyst
E. Involves up regulation of integrins
30. A 32-years old nulliparous woman has had oligomenorrhea since menarche. During the
past 5 years
she has experienced slow but progressive increase in hair on her face, back and forearms
in the
intermammary space, and on the back of her hands. Her voice has slowly deepened and
temporal
balding and clitoromegaly have developed. Which of the following is the most likely
diagnosis?
A. Polycystic ovary syndrome
B. Cushing’s syndrome
C. Stromal hyperthecosis
D. Ovarian tumor
E. Testis tumor
31. Fertility is reduced with increasing age of women because of a decreasing number of
oocyte biology
this is/are true. Except :
A. There is loss oocyte before birth
B. The total number of oocytes both declines bi-exponentially with age and the loss
accelerates
around the age of 37 years
C. Genetic factors are more important than environmental factors in determining
the decline in
the number of oocytes
D. Recent evidence suggest thet ovarian stem cells are present in humans and are
able to
replenish in the number of oocytes
E. Alterations in the neuroendocrine control with advancing age suggest a central
contribution
of the decline in the oocyte number
32. The known causes of reccurent miscarriage may be grouped as follows, expect :
A. Balanced translocation
B. Uterine septum
C. Toxoplasma infection
D. Bacterial vaginosis infection
E. Thrombophilia related
33. Polypeptide hormone which helps gonadotrophin actions in theca and granullosa cells is
A. Insulin-like growth factors
B. Anti mullerian hormone
C. Epinephrine
D. Glucagon
E. Leprin
34. In the management of adenomyosis, the following statements are true, except :
A. The LNG-IUS has been shown both shrink adenomyotic deposits and overall
improve
menstrual symptoms
B. Myometrial resection of adenomamyomata has been shown to improve
menstrual symptoms
in the majority of case series
C. In the presence of rectovaginal adenomyamata, extensive surgery including
hysterectomy is
ussually required
D. Succesful pregnancies have been reported after myometrial resection of
adenomyomata
E. During vaginal hysterectomy there appears to be a higher incidence of bladder
injury in the
presence of adenomyosis
35. The following are true regarding the induction of ovulation for woman with PCOS :
A. Laparascopic ovarian drilling is as effective as low dose gonadotrophin therapy
B. Laparascopic ovarian drilling is only effective for approximately six month
following the
operation
C. Co-treatment with GNRH agonist and low dose gonadotrophins reduces the risk of
OHSS
D. A pregnancy rate of approximately 80% is achieved with clomiphene citrate
within 12 cycles
E. The advantage of clomiphene over aromatase inhibitors is their lack of anti-
oestrogen action
on cervical mucous and endometrium
36. Which the following can be the etiology of either primary or secondary amenorrhea?
A. Asherman’s syndrome
B. Gonadal agenesis
C. Sheehan’s syndrome
D. Kallman’s syndrome
E. Anorexia nervosa
37. Which of te following statement regarding hypothalamic amenorrhea is true?
A. Patients with hypothalamic amenorrhea are hypogonadotropic-normogonadsm
B. Patients with hypothalamic amenorrhea are hypogonadotropic-hypogonadsm
C. Patients have a deficiency in SHBG secretion
D. Nearly always be anorexia and weight loss
E. Patients can be treated by clomiphene citrate

38. Which regard to endometriosis and infertility. The following statements are true, except:
A. It is generally accepted that the laparascopic surgical treatment of minimal or
mild
endometriosis increases fecundity rate
B. For moderate or severe endometriosis, results after laparascopy are comparable
to those
after laparatomy
C. The endometrioma in most cases is a pseduocyst
D. The consesnsus is that laparascopic surgical treatment of mild endometriosis
restores fertility
to normal
E. There is ample evidence to suggest that ovarian follicle are present at the base of
the
endometriotic pseudocyst
39. The following condition is not a diagnostic criteria of PCOS according to Rotterdam
criteria :
A. Polycystic ovary appeareance on sonographic examination
B. Exclusion of another type of hyperandrogenism
C. Oligoovulation
D. Hirsutism
E. Obesity
40. Anovulation which occur after ovaritoxic agent chemotherapy was classified as :
A. WHO anovulation class 1
B. WHO anovulation class 2
C. WHO anovulation class 3
D. WHO anovulation class 4
E. WHO anovulation class 1 and 3
41. The increase of insulin level in PCOS may lead to hyperandrogenism due to :
A. Decrease of SHBG
B. Increase of SHBG
C. Decrease of androgen
D. Decrease of SHBG elimination
E. Decrease of free androgen
42. Synergistically, insulin wil cooperate with following hormone to increase androgen
production in
PCOS :
A. LH
B. FSH
C. Inhibin B
D. Inhibin A
E. Estradiol
43. In weight loss management in PCOS, the most important aspect is :
A. Overall caloric restriction
B. Carbohydrate restriction
C. Fat rsetriction
D. Activity restriction
E. Appetite restriction

44. The future medical condition which may occur due to PCOS :
A. Cervical malignancy
B. Endometrial malignancy
C. Osteoporosis
D. Alzheimer disease
E. Parkinson disease
45. If a PCOS woman still not ovulating after life style modification, weight loss, and
clomiphen citrate
administration, the next step will be :
A. Metformin administration
B. Progestin administration
C. Gonadotrophin administration
D. Aromatase inhibitor administration
E. Combined oral contraception administration
46. In PCOS, the consequence serine phosphorylation disturbance is :
A. Increase of Akt pathway activation
B. Disturbance of Akt pathway
C. PI3K activity is increased
D. Disturbance of insulin binding to its receptor
E. Decreased testosterone production
47. The evidence of genetic factor in PCOS is:
A. 50% increase of PCOS incidence in siblings
B. Patients of PCOS patient did not reveal any insulin resistence conditions
C. PCOS patients have lower average birthweight
D. PCOS patients have a hystory of maternal consumption of particular drugs
E. PCOS patients have hystory of abnormal nurturing during their childhood period
48. The gold standart of insulin resistence measurement is :
A. QUICK1 method
B. HOMA-IR method
C. Anthropometric method
D. Euglycemic clamp method
E. Oral glucosa tolerance test method
49. Metformin therapy for PCOS correlated with :
A. Increase of body mass index
B. Increase of insulin sensitivity
C. Increase of blood pressure
D. Increase of LDL level
E. Increase of fasting blood glucosa level
50. DHEA mostly produced by :
A. Adipose tissue
B. Adrenal tissue
C. Ovary
D. Hypophysis
E. Liver

51. The important enzyme for androgen potency is :


A. 17hidroxysteroid dehydrogenase
B. Estradiol 2-hydroxylase
C. 5 alpha reductase
D. Tyrosin kinase
E. Aromataase
52. Even after menopause most woman have circulating estrogen. It mainly originates from
the
aromatization of :
A. Androstenedione to estrone by ovarian granulosa cells
B. Androstenedione to estrone by ovarian theca cells
C. Androstenedione to estrone by adipose tissue
D. Testosterone to estradiol by adipose tissue
E. Estradiol to estrone by adipose tissue
53. Estrogens are produced by the mother, fetus, and placenta. Which one of the following
is true?
A. Estradiol accounts for 80% of the estrogen produced during pregnancy
B. Estriol is produced primarily by the placenta
C. Anencephaly is associated with a normal level of estriol
D. Estrogen supresses oxytocin secretion
E. Estrone accounts for 80% of the estrogen produced during pregnancy
54. The decrease of progesterone in the end of menstrual cycle will lead to :
A. The decrease of plasminogen activator inhibitor-1 activity
B. The activation of metalloproteinases
C. The blocking of metalloproteinase
D. The decrease of prostaglandin activity
E. The decrease of inflammation activity
55. The example of estrogen withdrawal bleeding is :
A. Corpus luteum regression
B. Withdrawal of progestin pills
C. The excessive effect of progestin
D. Post-oophorectomy in follicular phase
E. Estrogen dominance in chronic anovulation
56. The laparascopic ovarian drilling is the choice of ovulation induction method PCOS
patient if :
A. The basal LH level more than 10 mIU/ml
B. Very low basal estradiol level
C. Antiestrogen administration failed
D. No ovulation after metformin administration
E. Polycystic ovary morphology was seen with ultrasonography
57. Hyperprolactinemia may caused anovulation throught :
A. Increase of continuous GnRH secretion
B. Increase of inhibin activity in target organ
C. Block the estrogen effect in target organ
D. Block the FSH effect in target organ
E. Block the GnRH pulsatile secretion

58. The abnormal uterine bleeding due to coagulophaty may only on seen as :
A. Spotting
B. Gum bleeding
C. Postpartum bleeding
D. Excessive bleeding since menarche
E. Excessive bleeding during progestin contraception administration
59. In abnormal uterine bleeding, biopsy was not cosidered necessary if the endometrial
thickness was:
A. 5-12 mm
B. >12 mm
C. >14 mm
D. < 8 mm
E. < 4 mm
60. In unopposed estrogen condition, progestin therapy would reduce the severity of
abnormal uterine
bleeding throught mechanism :
A. Inducing aromatase activity
B. Inducing 5-alpha reductase activity
C. Inducing estrogen receptor formation in endometrium
D. Inducing 17-beta hydroxysteroid dehydrogenase and sulphotransferase to
convert estradiol
to estrone
E. Inducing 17-beta hydroxysteroid dehydrogenase and sulphotransferase to convert
estrone to
estroadiol
61. The benefit of NSAID in abnormal uterine bleeding is :
A. No interference to ovulation
B. Increase the prostacyclin level
C. Work in hypophysis-ovary axis
D. Can be applied in patient with coagulopathy
E. May lead to regular menstrual cycle
62. The indication for GnRH agonist therapy in management of uterine fibroids is :
A. May allow for anemia correction in patient with severe anemia
B. Remove the fibroid mass due to estrogen deprivation
C. Reduce the endometrial thickness in unopposed estrogen condition
D. Reduce the amount of blood transfusion during operation
E. Avoiding the endometrial atrophy
63. With regard to strategies for preventing OHSS, the following statements are true,
except :
A. The use of the GnRH agonist long down regulation protocol is associated with a
higher
incidence of OHSS
B. GnRH antagonist protocols do not cause even more severe OHSS
C. Serum oestradiol levels are closely and positively correlated to OHSS, and can be
used in
isolation to predict OHSS
D. Leutinizing hormone can be used in place of hCG to reduce the risk of OHSS
E. Measeres to reduce the risk of OHSS include “coasting”, withholding hCG, early
hCG
administration and freezing all embryos

64. The following is true about hyperinsulinemia in women with PCOS, except :
A. Hyperinsulinemia is common in obese women but does not occur in lean women
B. It has been attributed to abnormal serine phosphorylation of the insulin receptor
C. Hyperinsulinemia is more commonly associated with infertility than any other
endocrine/metabolic factor
D. Hyperinsulinemia stimulates ovarian androgen production
E. Hyperinsulinemia is associated with an increased risk of early pregnancy loss
65. The following immunological function(s) are/is suppressed in women with endometriosis
:
A. Cytokine synthesis
B. Antibody synthesis
C. NK function
D. Phagocytosis
E. Leukocyte chemotaxis
66. The following are considered a mandatory part of investigation for menorrhagia :
A. Full blood count
B. Serum ferritin
C. Coagulation screen
D. Thyroid function
E. Endometrial biopsy
67. The indication for estrogen therapy in abnormal uterine bleeding is :
A. Bleeding due to chronic anovulation
B. Bleeding due to coagulation defect
C. Intermenstrual bleeding during progestin therapy
D. Bleeding due to endometrial polyp
E. Bleeding due to local inflammation from the IUD
68. PIBF, a protein mediating the immuno-modulatory effects of progesterone during
pregnancy is
known to, except :
A. Induce the pregnancy-protective shift from Th2 dominance to Th1 dominance
B. Advance the synthesis of a asymmetric antibodies
C. Increase during normal pregnancy from 7 th to 37th weeks
D. Supress NK cell activity
E. Be produced extensively after organ transplantation
69. The following are characteristics of embryos at the blastocyst stage, except :
A. This stage is reached at 5 or 6 days after fertilisation of the egg
B. They are developing under the control of the embryonic genome
C. They generally have higher implantation rates than cleavage stage embryos
following in vitro
fertilisation
D. After fertilisation in vivo they are typically located in the fallopian tube
E. In vitro culture of embryos to this dtage preferably involves use of sequental
culture media
70. Hyperprolactinemia can cause secondary amenorrhea by :
A. Inhibiting activity by interacting with hypothalamic dopamine and opioidergic
systems
B. Directly inhibiting ovarian activity
C. Causing galactorrhea
D. Increasing gonadotropin production
E. Increasing inhibin production
71. Appropriate therapy for hyperinsulinemic women with polycystic ovary syndrome
includes :
A. Bromocriptine
B. Troglitazone
C. Metformin
D. Clomiphene citrate
E. Oophorectomy
72. In the treatment of women with PCOS which of the Following is true :
A. The use of the combined oral contraceptive pill for the amelioration of the
androgenic
symptoms of PCOS is of greatest benefit to lean woman
B. Firm evidence exist that early pregnancy treatment with metformin reduces the
risk of
miscarriage in women with PCOS
C. Metformin does not cross the placenta and is therefore considered safe in
pregnancy
D. Metformin is more beneficial in the treatment of hirsutism in women with PCOS
than
sprinolactone
E. Ovarian drilling (diathermy) is as effective long-term as gonadotrophin therapy at
inducing
ovulation in clomiphene resistent anovulatory women with PCOS
73. A 16 year old female with primary amenorrhea is tanner stage 1 for both breast and
public
development. Serum FSH and LH levels are both elevated. Which of the following studies
is indicated
at this time :
A. Dihydroepiandrosterone sulfate (DHEAS) level
B. Testosterone level
C. Computed tomography of the abdomen
D. Chromosomal analysis
E. Prolactin level
74. The abnormal uterine bleeding due to unopposed estrogen could be found :
A. PCOS
B. Low BMI
C. Endometriosis patient
D. Low ovarian reserve patient
E. Post-oophorectomy patient
75. The estrogen therapy may be of benefit in the conditions :
A. Breakhtrough bleeding in COC users
B. PCOS related bleeding
C. Endometrial hyperplasia is related bleeding
D. Endometrial polyp related bleeding
E. Chronic anovulation related bleeding
76. The management of abnormal uterine bleeding may consist of :
A. Administration of copper-IUD
B. Endometrial biopsy
C. Administration of dopamine agonist drug
D. Administration of calcium
E. Administration of alendronate

77. The feature may be correlated with PCOS :


A. Low insulin resistance
B. Low level of DHAS
C. Low level of future risk of cardiovascular disease
D. Low SHBG
E. Low estradiol
78. The drug which may have bone-losing effect is :
A. GnRH agonist
B. Estrogen
C. Calcium supplement
D. Exogenous FSH
E. Exogenous hCG
79. A-28 year old nulliparaous woman request evaluation for hirsutism. The hair on her face,
chest and
back is of the fine, lanugo type. She has central obesity, abdominal striae, peripheral
muscle wasting
and dorsal neck fat pads. Which of the following would be the best laboratory test for
evaluation of
this patient :
A. Serum testosterone
B. Overnight dexamethasone suppression test
C. Serum dehydroepiandrosterone sulfate
D. Serum ACTH
E. Serum estrogen
80. A patient present with amenorrhea and galactorrhoea. Her prolactin levels are elevated.
She is not
and never has been pregnant. In additional to evaluating her for a prolactinoma, one also
needs to
evaluate for other causes that would increase prolactin such as elevated :
A. Corticotrophin-releasing hormone
B. Dopamine
C. Gamma-aminobutyric acid (GABA)
D. Throid stimulating hormone
E. Thyrotropin-releasing hormone (TRH)
81. A 32-year old nulliparous woman has had oligomenorrhea since menarche. During the
past 5 years
she has experienced slow but progressive increase in hair on her face, back and forearms
in the inter
mammary space, and on the back of her hands. Her voice has slowly deepened and
temporal balding
and clitoromegaly have developed. Which of the following is the most likely diagnosis :
A. Polycystic ovary syndrome
B. Cushing’s syndrome
C. Stromal hyperthecosis
D. Ovarian tumor
E. Testis tumor
82. Which of the following statement regarding hypothalamic amenorrhea is true :
A. Patient with hypothalamic amenorrhea are hypogonadoytropic-normogonadism
B. Patient with hypothalamic amenorrhea are hypogonadoytropic-hypogonadism
C. Patients have a deficiency in SHBG secretion
D. Nearly always be anorexia and weight loss
E. Patients can be treated by clomiphene citrate
83. Since the work up of an elevated prolactin level can be expansive, it is appropriate to
draw prolactin
levels when the lowest values are to be except :
A. Decrease sshortly after sleep
B. Increase after ingesting high-carbohydrate meals
C. Increase during stress
D. Decrease during surgery
E. Decrease after excercise
84. Which of the following statements regarding GnRH-stimulated LH secretion is accurate :
A. It is enhanced by gonadotrope exposure to progesterone
B. It is enhanced by gonadotrope exposure to testosterone
C. It is enhanced by gonadotrope exposure to continuous GnRH
D. It is enhanced by gonadotrope exposure to estrogen
E. It is associated with steady LH release
85. The progressive sequence (with steps omitted) in the metabolism of steroid hormone is :
A. Cholesterol-estradiol-testosterone-pregnolone
B. Cholesterol-pregnolone-cortisol-estradiol
C. Cholesterol-pregnolone-estrone-androstenedione
D. Cholesterol-androstenedione-pregnolone-estradione
E. Cholesterol-pregnolone-androstenedione-estrone
86. All drug for medical treatment of hirsutism should seek to alter at least one of the major
aspect of
androgen metabolism, except :
A. Decrease production
B. Inhibit androgen receptor
C. Increase SHBG
D. Decrease the metabolic clearance rate
E. Inhibit peripheral enzyme production
87. The New progestin
A. Demonstrate decreased androgenicity
B. May increase insuline resistance
C. Decrease SHBG
D. Demonsrate increase mineralocorticoid activity
E. Cause markedly less breakthrough bleeding than traditional progestins
88. The use of contraceptive :
A. Protects againts endometrial cancer
B. A. Protects againts malignant breast cancer
C. A. Protects againts thromboembolic desease
D. A. Protects againts cardiovascular desease
E. A. Protects againts cerebrovascuar desease
89. The dominant follicle :
A. Has a lesser content of FHS receptors
B. Contains granulosa cells with greater rate of proliferation than its cohorts
C. Containts low concentration of local autocrine a paracrine peptides
D. Is selected ...
E. Is always........
90. Inhibin :
A. Is produced by FSH stimulated granulosa cells
B. Secretion is not inhibited by GnRH
C. Exist as only one isoforms
D. Rises throught the follicular phase to reach a midcycle peak followed by a greater
mildluteal
peak
E. Converted to activin by theca cells
91. The pituitary secretion of FSH is influenced by the balance of :
A. IGF secretion
B. Aromatase secretion
C. Progesterone secretion
D. GnRH secretion
E. Testosterone secretion
92. The role of androgen in folliculogenesis
A. At high concentration enhance their own aromatization and contribute to
estrogen
production
B. Increase the FSH receptor content of the follicle
C. At higher level cause follicular atresia
D. May serve as substrate for progesterone
E. always produced by adrenal gland
93. The preovulatory follicle :
A. As it approaches maturity produces decreasing amounts of estrogen
B. Contains granulosa cells which enlarge acquire lipid inclusions and theca which
becomes vacuolated and vascular
C. Does not produce progesterone prior to ovulation
D. Containts an oocyte which resumes mitosis
E. Laways 18 mm in diameter
94. Ovulation
A. Is most reliably predicted by the onset of the LH surge which occurs 24-36 hours
prior to
response
B. Occurs approximately 10-12 hours after the estradiol peak
C. Requires a threshold of LH conventration to be maintained for prolonged period
inorder for
full maturation of the oocyte to occur
D. Involves no variation in timing from cycle to cycle within the same women
E. Always occur in early morning
95. The enzyme that take an important role in endometrial shedding
A. Metalloproteinase
B. Aromatase
C. 5-alpha reductase
D. 17-hidroxysteroid dehydrogenase
E. Tyrosin kinase

96. The classification system used of abnormal uterine bleeding is :


A. ASRM classication system
B. ENZIAN system
C. PALM COEIN system
D. ESGE system
E. Rotterdam system
97. Amenorrhea due to structures exercise is classified as
A. Primary amenorrhea
B. Drug .... Amenorrhea
C. Ovarian failure amenorrhea
D. Galactorrhea amenorrhea
E. Hypothalamic amenorrhea
98. In anovulatory women, if the hormonal assay result is normal, then it classified as :
A. Group I anovulation
B. Group II anovulation
C. Group III anovulation
D. Group IV anovulation
E. Combined Group II and IV anovulation
99. Clomiphene citrate only can be used in
A. Group I anovulation
B. Group II anovulation
C. Group III anovulation
D. Group IV anovulation
E. Combined Group II and IV anovulation
100. Amenorrhea which occur in woman who already had menarche is called :
A. Primary amenorrhea
B. Drug induce amenorrhea
C. Ovarian failure amenorrhea
D. Galactorrhheic amenorrhea
E. Secondary amenorrhea

SOAL UNAS 1 23 MARET 2019


1. The complete set of DNA in an organism is called:
a) A chromosome
b) A gene
c) A proteome
d) A genome
e) A nucleus

2. A contiguous region of DNA that can encode a protein


product and contains within its regulatory sequences that
regulate its expression is called:
a) A genome
b) A gene
c) A chromosome
d) A proteome
e) A nucleus

3. A package of genetic material, consisting of a DNA


molecule to which are attached large numbers of proteins
that maintain chromosome structure and play a role in
gene expression is calle
a) A chromosome
b) A genome
c) A gene
d) A proteome
e) A nucleus
4. The biologic effects of major steroid hormones are
determined mainly by:
a) Their affinity to albumin
b) Their clearance rate
c) Their production rate
d) Their unbound portion
e) Their capability to enter reservoir organs

5. The major androgen products of the ovary are:


a) Androstenedione and testosterone
b) Dehydroepiandrosterone sulfate and androstenedione
c) Testosterone and dihydrotestosterone
d) Dehydroepiandrosterone sulfate and dihydrotestosterone
e) Androstenedione and dihydrotestosterone

6.In the major sex-steroid receptors, the role of heat


shock protein (HSP) is:
a) To enhance the binding potential of the receptor,
therefore prevent early separation with hormone
b) To stabilize the clearance rate of the hormone-receptor
complex and ensure the biological effects is optimally
achieved
c) To stabilize and protect the receptor and maintain
the DNA binding region in inactive state
d) To prevent the downregulation effect due to excessive
exposure from the hormone
e) To enhance the translation process in the DNA binding
region.
7. The major factor in the potency differences among the
various estrogens (estrone, estradiol, estriol) is:
a) The length of time the estogen is bound with the SHBG
b) The length of time the estrogen-SHBG complex cleared
from the circulation
c) The length of time of the estrogen-receptor complex
occupies the ribosome to synthesize proteins
d) The length of time of the estrogen-receptor complex
occupies the nucleus
e) The length of time of dissociation between estrogen and
SHBG

8. The conformational changes of steroid receptors is


induced by:
a) Steroid hormone and lead to dissociation of HSP90
b) HSP90 and lead to the inactivation of steroid receptors
c) Steroid hormone and lead to the formation of dimers
without dissociating HSP90
d) TAF2 and may be achieved without full hormone binding
e) TAF1 and may be achieved without full hormone binding

9. The main characteristics of non-genomic, extranuclear


estrogen stimulation activity is:
a) Gene expression and lead to specific protein synthesis
b) Rapid response and associated with growth factors
and G-protein messenger
c) Classic dimerization after ligand binding
d) Gradual response involving receptor and ligand binding,
then activation of intracellular organelles
e) Activation of the estrogen receptors pathway through
second messenger involving phosphorylation of estrogen
receptor and coregulator proteins

10. The characteristics of gonadotrophin secretion in


childhood period is:
a) High level of gonadotrophin with maximal hypothalamic
suppression
b) Low level of sex steroid hormone with maximal
hypothalamic suppresion
c) Low level of gonadotrophin with no hypothalamic
suppression
d) Low level of gonadotrophin with maximal hypothalamic
suppression
e) High level of sex steroid hormone with no hypothalamic
suppresion
11. The following reactions are occur as consequences of
estrogen and progesterone withdrawal:
a) The process of regeneration, vasomotor reactions, and
tissue growth
b) The process of necrosis, vasomotor reactions, and
tissue loss
c) The process of regeneration, tissue loss, and vascular
constriction
d) The process of apoptosis, vasomotor reactions, and
tissue loss
e) The process of apoptosis, tissue regeneration, and
vascular breakdown
12. The following substances are induced after estrogen
and progesterone withdrawal in menstrual cycle:
a) IGF-I
b) Aromatase
c) DNA transcriptase
d) Metalloproteinase
e) Inhibin A

13. Prolactin secretion is mainly controlled by:


a) Stimulatory control of GnRH
b) Inhibitory control of hypothalamic dopamine
c) Stimulatory control of TRH
d) Inhibitory control of kisspeptin
e) Inhibitory control of ACTH

14.The short feedback loop in neuroendocrine system


means:
a) A negative feedback of pituitary hormones on their
secretions, presumably via inhibitory effects on releasing
ho
b) A feedback effects of circulating effects of target gland
hormones
c) A negative feedback from adjacent cells in the same
tissue
d) A negative feedback of pituitary hormones on their
secretions, presumably via inhibitory effects on
releasing hormones in the hypothalamus.
e) A negative feedback from endocrine gland cells to their
neighboring glands
15. Biomolecular technique which allows one small
portion of DNA to be replicated and analyzed is:
a) Polymerase Chain Reaction
b) Southern Blot Analysis
c) Hybridization
d) Cloning DNA
e) Northern blotting

16. Polypeptide hormone which helps gonadotrophin


actions in theca and granulosa cells is:
a) Insulin-like growth factors
b) Anti-mullerian hormone
c) Epinephrine
d) Glucagon
e) Leptin

17. The important enzym for androgen potency is:


a) 17 hidroxysteroid dehydrogenase
b) estradiol 2-hydroxylase
c) tyrosin kinase
d) 5 alpha reductase
e) aromatase
18. The decrease of progesterone in the end of menstrual
cycle will lead to:
a) the activation of metalloproteinases
b) the decrease of plasminogen activator inhibitor-I activity
c) the blocking of metalloproteinases
d) the decrease of prostaglandin activity
e) the decrease of inflammation activity

19. The major function of zona pellucida during


fertilization is:
a) A selective barrier for every sperm to protect from
premature fertilization
b) Contains ligand for sperm which is species-specific
c) Contains antibody for sperm to protect multiple
fertilization
d) A nonselective chemoattractants for sperm
e) Contains specific proteins to protect from oocyte
postmaturity

20. The most optimal timing of endometrial receptivity is


characterized by 
a) Sperm-induced cortical reactions
b) Estrogen induced spiral artery remodellings
c) Progesterone-induced pinopode formations
d) Sperm-induced acrosomal reactions
e) Inhibin-B induced FSH decrease

21. The function of pinopodes is:


a) To provide the wide surface for blastocyst implantation
b) To enhance endometrial fluid production to promote
embryo implantation
c) To create more vascular area to promote more cytokine
production
d) To absorb the fluid and forcing blastocyst to be in
contact with the endometrium
e) To block the cortical reactions therefore prevents
polyspermia reactions.

22. The critical features of transition of estrogen


suppression to LH stimulation in the midfollicular phase
are:
a) The estradiol bind by SHBG and hepatic clearance
b) The estradiol production rate and hepatic clearance rate
c) The estradiol concentration and duration of
sustained elevation
d) The estradiol concentration and the rate of SHBG
conjugation
e) The estradiol production rate and its affinity with their
receptors in granulosa cells

23. The characteristics of gonadotropin pulsation during


follicular phase are:
a) Less frequent and greater in amplitude
b) More frequent but smaller in amplitude
c) More frequent and greater in amplitude
d) Less frequent and smaller in amplitude
e) More frequent throughout the cycle with logarithmic
amplitude

24. The majority of follicular development time until


reaching the ovulatory stage was spent on
a) Gonadotrophin dependent stage
b) Cyclic recruitment stage
c) Gonadotrophin independent stage
d) Domination stage
e) Resting pool stage

25. The continuing growth of antral follicles in response


to FSH is known as:
a) Selection stage
b) Domination stage
c) Recruitment stage
d) Final maturation stage
e) Atresia stage

26. The main function of AMH in the follicular


development is:
a) To promote granulosa cell differentiation
b) To inhibit primordial follicel growth
c) To stimulate the follcle responsiveness to FSH
d) To stimulate the follicle responsiveness to LH
e) To induce final oocyte maturation

27. The FSH rise in few days before menses is triggered


by:
a) teep rise of midcycle steroidogenesis
b) A decline in luteal phase steroidogenesis and inhibin
A secretion
c) The sustained high level of progesterone during luteal
phase
d) The decline of ovarian steroid after LH surge is achieved
e) The activation of inhibin B in midfollicular phase

28. The rate of androgen conversion to estrogen in the


granulosa cells is limited by:
a) The aromatase activity
b) The long negative feedback of estrogen to hypothalamus
c) The intracrine regulations by growth factors
d) The presence of abundance cholesterol as basic
substrate
e) The maturity and differentiation of theca cells

29. In the presence of FSH, the dominant substance in


the follicular fluid is:
a) Androgen
b) Amino acids
c) Estrogen
d) Growth factors
e) Cumullus cells

30. During the folliculogenesis, if plasma LH is


prematurely elevated in the follicular phase, it may lead
to:
a) The decrease of granulosa cell mitotic activity
b) The rise of intrafollicular estrogen
c) The rise of intrafollicular growth factors
d) The decrease of intrafollicular androgen
e) The rise of long positive feedback to estrogen activity

31. The selection of dominant follicle is the result of:


a) The effect of estrogen on pituitary secretion of FSH
b) The effect of estrogen on granulosa cells actions of
growth factors
c) The effect of production of FSH receptors in the
follicle
d) The effect of proliferation of LH receptors in the follicle
e) The effect of estrogen on surrounding granulosa cells

32. The highest concentration of AMH was found in:


a) Atretic follicles
b) Small antral follicles
c) Dominant follicles
d) Early primordial follicles
e) Preovulatory follicles

33. The serum AMH level can be accurately measured in


any day of menstrual cycle because of:
a) It is secreted steadily by all follicular cells
b) It is stimulated by high level of ovarian steroids in the
midcycle period
c) It is stimulated by GnRH throughout the cycle
d) It is not affected by gonadotrophins or sex steroids
e) It is not affected by the rate of atresia in every cycles
34. During periovulatory period, LH surge will trigger
plasminogen activators produced by granulosa cells will
activate plasminogen to plasmin in follicular fluid to
activate:
a) Progesterone synthesis
b) Collagenase system
c) Androgen synthesis
d) Long feedback system
e) Second meiosis
35. The mechanism that may limit the duration of LH
surge is:
a) The decrease of midcycle FSH level
b) The progressive rise of estrogen via negative feedback
c) The decrease of inhibin-B secretion
d) The progressive rise of progesterone via negative feedback
e) The intrafollicular rise of insulin-like growth factor II

36. This drug may inhibit the ovulation process in the


ovulatory period:
a) Dinoprost
b) Folic acid
c) Oxytocin
d) Ketoprofen
e) Cholecalciferol

37. The lifespan and steroidogenic capacity of the corpus


luteum are dependent on:
a) Continued tonic LH secretion
b) Positive feedback of preovulatory estrogen rise
c) Continued tonic FSH secretion
d) Negative feedback of luteal phase progesterone to
hypothalamus
e) Continued rise of inhibin-B secretion
38. During the luteal phase, the initiation of new
follicular growth is inhibited by:
a) The low level of of ovarian steroids due to demise of the
corpus luteum
b) The low level of gonadotrophin due to negative feedback of
estrogen, progesterone, and inhibin-Ac.
c) The declining level of LH immediately after its surge
d) The declining level of estrogen before LH surge is reached
e) The increasing level of FSH in the luteofollicular transition
39. The important aspect of corpus luteum rescue by
hCG is to:
a) Suppress tissue inhibitors of metalloproteinases (TIMP)
b) Increase the aromatase expression
c) Prevent matrix metalloproteinase expression
d) Increase plasmin expression
e) Prevent steroidogenesis synthesis
40. The process of nuclear division in all somatic cells is
called 
a) Meosis
b) Mitosis
c) Prophase
d) Anaphase
e) Telophase

41. Acrosome reaction produces:


a) Separation between head and tail of spermatozoa
b) Expansion of cumullus cells
c) Separation between midpiece and tail of spermatozoa
d) Release of enzymes such as hyaluronidase and acrosin
e) Binding of spermatozoa with zona pellucida

42. Zone reaction will produce:


a) Zona pellucida become impermeable after fertilization of
one spermatozoa
b) Zona pellucida become permeable to allow easy penetration of
spermatozoa
c) Zona pellucida expand to allow easy penetration of
spermatozoa
d) Zona pellucida become thin to allow easy penetration of
spermatozoa
e) Zona pellucida become thin to produce dense layer of
cumullus cells

43. Capacitation is characterized by:


a) Mitochondrial reaction, binding capabilities with cumullus,
hypomotility
b) Mitochondrial reaction, binding capabilities with zona
pellucida, hypomotility
c) Acrosomal reaction, binding capabilites with zona
pellucida, hypermotility
d) Acrosomal reaction, binding capabilites with cumullus,
hypermotility
e) Acrosomal reaction, binding capabilites with zona pellucida,
hypomotility

44. The role of progesterone in pregnancy is:


a) Fetal cholesterol pool
b) Substrate for glucocorticoid and mineralocorticoid
production in fetal adrenal gland
c) Substrate for estrogen production in placental compartment
d) Substrate for androgen production in maternal compartment
e) Maternal cholesterol pool

45. Rapid conjugation of sex steroid hormone with


sulfate compound is:
a) To protect the fetus from adverse effect of abundant sex
steroid hormone
b) To convert to more active form of sex steroid hormone
c) To ensure the placental sulfatase enzyme in inactive condition
d) To change to longer action form of steroid hormone
e) To protect the maternal compartment from sex steroid
hormone deficiency

46. Cortisol from maternal compartment cannot


influence the fetal development due to:
a) Its conversion to progesterone
b) Its conversion to aldosterone
c) Its conversion to cortisone
d) Its conversion to estrone
e) Its conversion to DHAS

47. The fetus is protected from masculinization due to


high level of androgen during pregnancy by:
a) Aromatization in maternal compartment and conjugation with
sulfate compound
b) Aromatization in placental compartment and conjugation
with sulfate compound
c) Aromatization in placental compartment and conjugation with
hydroxy compound
d) Aromatization in placental compartment and conjugation with
glucuronyl compound
e) Aromatization in fetal ovary compartment and conjugation
with hydroxy compound

48. The decrease of estrogen and progesterone level in


late luteal phase lead to:
a) Increase of inhibin A secretion
b) Decrease of pulsatile frequency secretion of GnRH
c) Increase of pulsatile frequency secretion of GnRH
d) Sustained feedback inhibition to hypothalamus until menstrual
phase
e) Sustained feedback inhibition to hypothalamus until follicular
phase

49. Polypeptide hormone which helps gonadotrophin


actions in theca and granulosa cells is:49. Polypeptide
hormone which helps gonadotrophin actions in theca and
granulosa cells is:
a) Anti-mullerian hormone
b) Epinephrine
c) Insulin-like growth factors
d) Glucagon
e) Leptin

50. The role of androgen in folliculogenesis:


a) At high concentration enhance their own aromatization and
contribute to estrogen production
b) Increase the FSH receptor content of the follicle
c) May serve as substrate for progesterone
d) At higher level cause follicular atresia
e) Always produced by adrenal gland

51. The preovulatory follicle


a) As it approaches maturity produces decreasing amounts of
estrogen
b) Contains granulosa cells which enlarge and acquire lipid
inclusions and theca which becomes vacuolated and
vascular
c) Does not produce progesterone prior to ovulation
d) Contains an oocyte which resumes mitosis
e) Always 18 mm in diameter
52. Ovulation
a) Is most reliably predicted by the onset of the LH surge
which occurs 24-36 hours prior to response
b) Occurs approximately 10-12 hours after the estradiol peak
c) Requires a threshold of LH concentration to be maintained for
prolonged period in order for full maturation of the oocyte to
occur
d) Involves no variation in timing from cycle to cycle within the
same woman
e) Always occur in early morning

53. The enzyme that take an important role in


endometrial shedding:
a) aromatase
b) 5alpha reductase
c) metalloproteinase
d) 17 hidroxysteroid dehydrogenase
e) tyrosin kinase
54. The estrogen during pregnancy influence:
a) Placental development
b) Progesterone production
c) Maternal blood pressure regulation
d) Fetal brain development
e) Fetal gender assignment

55. The substrate for fetal adrenal gland production of


glucocorticoids and mineralocorticoids is:
a) Androgen
b) Progesterone
c) Estrogen
d) Somatomammotropin
e) Prostaglandin
56. The circulating level of SHBG is inversely corelated
with:
a) Body height
b) Estrogen production
c) Body weight
d) Hypothalamic signaling
e) Long feedback to hypothalamus

57. The main difference between TAF-1 and TAF-2 is:


a) TAF-2 can stimulate transformation change without hormone
binding
b) TAF-1 is located in the hormone binding domain of estrogen
receptor
c) TAF-1 can stimulate transcription without hormone
binding
d) TAF-2 is located in the regulatory domain of estrogen receptor
e) TAF-1 need strong hormone binding to induce transcription
process

58. The “chaperone” protein that keeps the estrogen


receptor in inactive form is:
a) HSP 90
b) TAF-1
c) TAF-2
d) SHBG
e) IGF-II
59. Polypeptide hormone which helps gonadotrophin
actions in theca and granulosa cells is:
a) Anti-mullerian hormone
b) Epinephrine
c) Glucagon
d) Insulin-like growth factors
e) Leptin
60. FSH function in follicular phase is:
a) Avoid oocyte maturation inhibitor effect on oocyte
b) Avoid domination of developing follicle
c) Avoid premature lutenization of oocyte
d) Avoid apoptosis of follicle
e) Avoid multiple oocyte maturation

61. Estrogen produced by dominant follicle has an effect:


a) Stimulates negative feedback on LH
b) Inhibits positive feedback on LH
c) Stimulate FSH receptor formation in dominant follicle
d) Inhibits testosterone aromatization on granulosa cell
e) Inhibit LH receptor formation in dominant follicle

62. The basic philosophy of pulsatile gonadotrophin


secretion is:
a) To avoid downregulation
b) To avoid positive feedback effect
c) To avoid negative feedback effect
d) To avoid accumulation of hormone in target tissue
e) To avoid degradation of hormone in bloodstream
63. Estrone to estradiol conversion is facilitated by:
a) 3 beta hydroxysteroid dehidrogenase
b) 17 beta hydroxysteroid dehidrogenase
c) P450 side chain cleavage
d) P450 aromatase
e) 5 alfa reductase

64. LH surge incites following events:


a) Meiosis, theca cell luteinization, theca cell expansion, OMI
synthesis
b) Meiosis, theca cell luteinization, cumulus expansion, OMI
inhibition
c) Meiosis, granulosa cell luteinization, cumulus expansion,
prostaglandin synthesis
d) Meiosis, granulosa cell luteinization, theca cell expansion,
luteinization inhibitor synthesis
e) Meiosis, granulosa cell luteinization, granulosa cell expansion,
prostaglandin inhibition

65. The rise of FSH level in periovulatory period has


consequence:
a) Bind oocyte to its attachment in follicle, convert plasminogen
to plasmin, FSH receptor formation
b) Bind oocyte to its attachment in follicle, convert plasminogen
to prostaglandin, FSH receptor formation
c) Release oocyte from its attachment in follicle, convert
plasminogen to prostaglandin, LH receptor formation
d) Release oocyte from its attachment in follicle, convert
plasminogen to plasmin, LH receptor formation
e) Release oocyte from its attachment in follicle, convert
plasminogen to plasmin, prostaglandin receptor formation

66. The loss of oocytes during atresia is done through:


a) Necrobiosis process
b) Necrosis process
c) Inflammation process
d) Apoptosis process
e) Entosis

67. The characteristics of secretory phase endometrium


is:
a) Continue to grow after ovulation
b) Progressive tortuosity of the endometrial glands
c) Increase in mitosis
d) Increase in DNA synthesis
e) Progressive decrease of estrogen receptors

68. These events is caused by high progesterone level in


the endometrium:
a) Prostaglandin and VEGF expression
b) Perivascular growth and decidualization
c) Cytokine and MMP expression
d) Leukocyte infiltration and vasoconstriction
e) Activation of caspase and vasodilatation

69. These events is caused by progesterone withdrawal in


the endometrium:
a) Suppression of prostaglandin and MMP expression
b) Suppression of caspase activities
c) Perivascular growth and gland proliferation
d) Increase in MMP and tissue breakdown
e) Reepithelization of endometrium

70. The menstrual flow stops as a result of:


a) Progesterone induced healing
b) Prolonged vasoconstriction of radial arteries
c) Activation of MMP systems
d) Autodigestion of endometrial glands
e) Activation of caspase enzymes

71. This condition is associated with the increase of


neuropeptide Y level:
a) Undernutrition
b) Obesity
c) Lack of sleep
d) Exposure of dark
e) Intermitten fasting

72. The suppression of gonadotrophins by GnRH agonist


can be used in the treatment of:
a) Polycystic ovary syndrome
b) Precocious puberty
c) Premenopausal syndrome
d) Premenstrual syndrome
e) Recurrent pregnancy loss

73. The GnRH antagonists can be used in the treatment


of:
a) Threatened abortion
b) Premenopausal syndrome
c) Premenstrual syndrome
d) Precocious puberty
e) Recurrent pregnancy loss

74. The characteristics of gonadotrophin secretion during


prepubertal period is:
a) Cyclic and showed different patterns between follicular and
luteal phase
b) Low but still demonstrate the irregular pulse
c) Sustained high level throughout the cycle
d) Sustained high level throughout the prepubertal period
e) Low and correlated with decreasing concentration of DHEA
75. The central inhibitory signal that restraint GnRH
pulsatile secretion is:
a) Prolactin
b) Endorphin
c) GABA
d) Dynorphin
e) Oxytocin
76. Progesterone production in pregnancy is independent
from:
a) Presence of fetus
b) Presence of placenta
c) Presence of fetal ovary
d) Presence of maternal ovary
e) Presence of fetal DHAS production

77. Hormones with specific binding protein are:


a) Thyroid and insulin
b) Thyroid and prolactin
c) Thyroid and epinephrin
d) Thyroid and testosteron
e) Thyroid and gonadotrophin

78. Hyperprolactinemia may caused anovulation through:


a) increase of continuous GnRH secretion
b) increase of inhibin activity in target organ
c) block the estrogen effect in target organ
d) block the FSH effect in target organ
e) block the GnRH pulsatile secretion

79. Which of the following factors is the most influence


for formation of the ext genitalia : 
a) Genetic sex of the embryo
b) A normal urogenital sinus
c) The sperm fertilizing the egg (carrying a Y chromosome)
d) The presence of fetal androgens
e) Maternal hormonal levels

80. The absence of the vagina is common in


a) Congenital adrenal hyperplasia in a female infant
b) Turner syndrome
c) Association with an absent or rudimentary uterus
d) Drug-induced fetal masculinization of a female infant
e) Gonadal dysgenesis

81. If germ cells fail to enter the developing genital


ridge, which of the following may occur?
a) Ovarian teratomas
b) Ectopic pregnancy
c) Ovarian choriocarcinoma
d) Gonadal agenesis
e) Testicular feminization

 82. The paramesonephric ducts will form 


a) The prostatic utricle
b) Seminal vesicles
c) Oviducts, uterus, and upper vagina
d) Upper vagina only
e) The ureters
83. The correct order of cell layers surrounding an
ovarian follicle from the oocyte outward is 
a) Zona pellucida, granulosa, theca interna
b) Granulosa, theca interna, zona pellucida
c) Theca interna, zona pellucida, granulosa
d) Theca interna, granulosa, zona pellucida
e) Zona pellucida, theca interna, granulosa

84. Most of the MHC antigens expression by trophoblast


are
a) Erithrocyte antigenic systems
b) HLA-G
c) uNK cells
d) Placenthal alkaline phosphatase
e) Rhesus-D

85. Which of the following changes will maintain the


corpus luteum ?
a) Decreased LH
b) Increased FSH
c) Increased hCG
d) Decreased inhibin B
e) Increased progesteron

86. This statement about GnRH secretion is correct,


a) The amplitude is highest at the early follicular phase.
b) Have lowest frequencies at luteal phase.
c) Have highest amplitude at late follicular phase.
d) Less frequencies at pre ovulatory than early follicular phase.
e) Lowest amplitude at luteal phase.

87. This statement is correct about Prolactin,


a) Produced by lobus posterior pituitary.
b) Produced only at the time of lactation.
c) Suppress GnRH secretion.
d) Stimulate LH production.
e) The production stimulated by dopamine.

88. This statement is correct about gonadotropin


production,
a) Is equal with serum gonadotropin level.
b) Stimulate by inhibin-A
c) Estrogen stimulate LH production.
d) Stimulate by low level serum progesterone.
e) Higher at mid luteal phase.

89. This statement is correct about serum gonadotropin


level,
a) Controlled by hypothalamus.
b) Stimulate by low level serum progesterone at early follicular
phase.
c) Suppressed by high level serum estrogen, at mid cycle.
d) Stimulate by activin.
e) Suppressed by inhibin-B

90. The role of LH at late follicular phase is,


a) Stimulate OMI production.
b) Stimulate granulose cells progesterone production.
c) Inhibit synthesis of progesterone.
d) Inhibit synthesis of inhibin-A
e) Suppress synthesis of androgen.

91. The requirement of serum estrogen level for getting


LH surge , minimally 
a) 100 pg/mL
b) 200 pg/mL
c) 300 pg/mL
d) 400 pg/mL
e) 500 pg/mL

92. The time of ovulation is:


a) 36 hours from the beginning of LH surge
b) 36 hours from the peak of LH surge
c) 36 hours from the beginning of FSH sure
d) 36 hours from the peak of FSH surge.
e) 36. Hours from the peak of mid cycle estradiol level

93. The growth of public and axillary hair at puberty is


due to an increased production of:
a) Gonadal androgens
b) Gonadotropins
c) GnRH
d) Estrogen
e) Adrenal androgen

94. Hormone that act on the cell they are secreted from,
are called :
a) Endocrine
b) Paracrine
c) Autocrine
d) Neurocrine
e) Exocrine
95. The primary genetic signal determining the direction
of gonadal differentiation in mammals:
a) SRY
b) TDF
c) XY
d) SF1
e) Akt-1

96. Vaginal epithelium and the fibromuscular wall of the


vagina originate from which of the following
respectively?
a) Mesonephric duct and endoderm of the urogenital sinus
b) Mesonephric duct and the uterovaginal primordium
c) Endoderm of the urogenital sinus and the mesonephric duct
d) Endoderm of the urogenital sinus and the urovaginal
primordium
e) Endoderm of the urogenital sinus and the
paramesonephric ducts
97. Which of the following is the principal site of
production of sex hormone-binding globulin (SHBG)?
a) Kidney
b) Pituitary
c) Liver
d) Hypothalamus
e) Granulosa cell

98. Follicle stimulating hormone (FSH) :


a) Is a steroid hormone
b) Is produced by the chromophil cells of the posterior lobe of the
pituitary gland
c) Stimulates the interstitial cells of Leydig in the male
d) Has alpha-subunits which are the product of the same gene
that controls the alpha-subunits of luteinizing hormone
(LH) and thyroid stimulating hormone (TSH)
e) Serum levels are decreased during the climacteric
99. Luteinizing hormone (LH) :
a) Is bound to plasma proteins
b) Secretion is stimulated by rising levels of oestrogen
c) Production is increased by sustained continuous administration
of gonadotrophin releasing hormone analogues
d) Inhibits testosterone production in the female
e) Surge occurs within 36 hours after ovulation

100. During growth of the embryo, the inner cell mass is


referred to as the embryonic disk. This embryonic disk
differentiates initially into
a) Decidua
b) Cytotrophoblast
c) Ectoderm and endoderm
d) Syncytiotrophoblast
e) Mesoderm

1. The mitosis is:


A. The process of nuclear division in gamet cells
B. The process of nuclear division in somatic cells
C. The process of nuclear division in oocytes
D. The process of nuclear division in spermatozoa
E. All of above is true
2. While circulating in the blood, 10–30% estradiol and testosterone is bound to a
protein carrier
A. Sex hormone binding globulin (SHBG)
B. Albumin
C. Corticosteroid binding globulin
D. A and B
E. Not all above

3. The major androgen products of the ovary are


A. Testosterone
B.Androstenedione
C.Dehydroepiandrosterone (DHA)
D. B and C
E. A, B, C are true

4. The excess genetic material of oocytes during each meiosis is extruded as


A. Cumulus oophorus
B. Barr’s body
C. Call-Exner body
D. Cytoplasm
E. Polar body

5. The cells that produce GnRH originate from


A. Optic area
B. Olfactory area
C. Oculatory area
D. A and B
E. Not all above

6. In the normal menstrual cycle corpus luteum secretes the following hormones,
except
A. Estriol
B. Progesterone
C. Inhibin-A
D. Androgen
E. Estradiol

7. The progressive sequence (with steps omitted) in the metabolism of steroid


hormone is
A. Cholesterol-estriol-testosterone-pregnenolone
B. Cholesterol-pregnenolone-cortisol-estradiol
C. Cholesterol-pregnenolone-estrone-androstenedione
D. Cholesterol-androstenedione- pregnenolone-estrone
E. Cholesterol-pregnenolone-androstenedione-estrone
8. Which of the following embryonic tissues contributes to the adult
fallopian tubes?
A. Coeloemic epithelium
B. Mesenchyme
C. Mesonephric duct
D. Paramesonephric duct
E. Urogenital ridge

9. Which of the following pubertal events is not mediated by gonadal


estrogen production
A. Breast development
B. Menstruation
C. Pubic hair growth
D. Skeletal growth
E. Vaginal cornification

10. Which of the following hormone(s) is / are produced by the corpus luteum ?
A. Progesterone only
B. Progesterone and estrogen only
C. Progesterone, estrogen, and inhibin only
D. Progesterone, estrogen, inhibin and relaxin only.
E. Progesterone, estrogen, inhibin, relaxin and contractin.

11. Steroid hormones are classified as:


A. Amino acids
B. Phospholipids
C. Lipids
D. Glycoprotein
E. None of the above

12. Management of anovulatory bleeding (dysfunctional uterine bleeding) depends


on the following conditions, except
A. Age of the patient
B. Desire for fertility
C. Size of the cervical os
D. Amount of bleeding
E. Cause of bleeding

13. Which of the following provide evidence of ovulation?


A. Day 14 progesterone level

B. Endometrial biopsy
C. Normal menstrual cycle.
D. Estrogen / progestogen ratio
E. LH / FSH ratio

14. Which of the following is not used to correct anovulation?


A. Bromocriptine
B. Weight loss
C. hCG injection
D. Low-dose estrogen
E. Clomiphene citrate
15. One statement below is associated with prolactin
A. Secreted by pituitary
B. Its secretion suppressed by TRH
C. Its secretion decrease at the time of lactation
D. Its secretion induced by light
E. Also secreted by placenta

16. One of the statements below is true about Mayer-Rokitansky-Kuster-


Hauser syndrome,
A. Lack of mullerian development
B. Presence of breast growth disturbance
C. Occurs dysfunctional uterine bleeding
D. Presence of sex hair growth disturbance
E. Demonstrate augmented gonadotropin secretion during sleep

17. Phytoestrogens are classified into


A. Black cohosh, genistein, red clover
B. Isoflavones, coumestans, and lignans
C. Bonestein, black cohosh, lignans
D. Isoflavones, red clover, bonestein
E. Not all above

18. Which one is the best predictor for the ovarian reserve ?
A. Basal FSH
B. Basal estradiol
C. Anti mullerian hormone
D. Inhibin B
E. All of above

19. Which of the following is consistent with a diagnosis of delayed puberty?

A. Breast budding in a 10-year-old girl

B. Menarche delayed beyond 16 years of age

C. Menarche 1 year after breast budding

D. FSH values less than 20 mIU/mL

E. Obese

20. Normal stature with minimal or absent pubertal development may be seen in

A. Testicular feminization

B. Kallman syndrome

C. Pure gonadal dysgenesis


D. Turner syndrome

E. Intermittent athletic training

21. Delayed puberty and sexual infantilism associated with hypergonadotropic hypogonadism
can be seen in patients with which of the following?

A. Adrenogenital syndrome (testicular feminization)

B. McCune-Albright syndrome

C. Kallman syndrome

D. Gonadal dysgenesis

E. Mullerian agenesis

22. Which of the following pubertal events in girls is not estrogen dependent?

A. Menses

B. Vaginal cornification

C. Hair growth

D. Reaching adult height

E. Production of cervical mucus

23. Which of the following findings characterizes a normal semen sample?

A. Agglutination

B. Sperm concentration of 35 million per mL

C. 5% normal sperm morphology

D. 10% progressive sperm motility

E. A volume of 1 mL

24. The dominant follicle is formed on day ..... of the menstrual cycle
A. 12 – 14
B. 9 – 11
C. 5–7
D. 2–4
E. 1 – 2

25. The following statements is true regarding the short loop feed back of GnRH.
A. Sex steroid relationship to pituitary.
B. Conection of GnRH with it’s own secretion.
C. Conection of hypothalamus to pituitary
D. With the down regulation mechanism
E. Through feed back of hypophyseal portal system.

26. The following is true regarding GnRH


A. The secretion is stimulated by norephineprin
B. Hypothyroid will stimulate it’s secretion.
C. Longterm and high dose of GnRH will increase multiple ovulation.
D. Pineal gland hormone stimulate it’s secretion
E. Oxytocin will increase it’s degradation.

27. A patient with PCOS often have an increase in insuline resistence. This will result
in an increase in
A. FSH.
B. Free estrogen level
C. Free testosterone level
D. Hepatic production of SHBG.
E. Progestogen levels.

28. Which of the following embryonic tissues contributes to the adult fallopian
tubes?
A. Coelemic epithelium
B. Mesenchyme
C. Mesonephric duct
D. Paramesonephric duct
E. Urogenital ridge.

29. Which of the following statements best discribes the role of FSH in
menstruation?
A. FSH stimulates follicle growth only in the early preanthral stage.
B. FSH increases its own receptor number on theca cells.
C. FSH induces theca cell aromatase.
D. FSH stimulates granulosa cell androgen production
E. FSH induces granulosa cell LH receptors within the dominat follicle.

30. During the evaluation for infertility, a woman may have an endomertial biopsy to
evaluate the quality of her ovulation since the development of the corpus luteum
is most closely associated with
A. Fertilization of an ovum
B. Follicular phase of the endometrium
C. Proliferative phase of the endometrium
D. Secretory phase of the endometrium
E. Sheding phase of the endometrium.

31. The three principal estrogens in women in decreasing order of potency are:
A. Estradiol, estriol, estrone
B. Estradiol, estrone, estriol
C. Estriol, estradiol, estrone
D. Estriol, estrone, estradiol
E. Estrone, estriol, estradiol

32. Which of the following is not used to correct anovulation?


A. Bromocriptine
B. Weight loss
C. HCG injection
D. Low-dose oestrogen
E. Clomiphen citrate

33. The following statements regarding prolactin is true :


A. It’s secreted by nucleus arcuatus
B. It stimulates oxytocn secretion
C. It stimulates GnRH secretion
D. The secretion is stimulated by TRH
E. It stimulates Norephineprin secretion

34. The following statements is true regarding the short loop feed back of GnRH.
A. Sex steroid relationship to pituitary.
B. Conection of GnRH with it’s own secretion.
C. Conection of hypothalamus to pituitary
D. With the down regulation mechanism
E. Through feed back of hypophyseal portal system.
35. The following is true regarding GnRH:
A. It’s a steroid hormone
B. It has membrane receptor
C. It enters posterior lobe of pituitary
D. It directly enters from tractus tubero infundibular.
E. The secretion is stimulated by dopamine.
36. The following is true regarding early relationship of hypothalamus-pituitary-ovarian axis
A. The first initiation is prior to puberty
B. Initiated with FSH increase.
C. At puberty, central hypersensitivity to estrogen is decline.
D. Soon will lead to regular ovulation.
E. Resulting in the first folliculogenesis.

37. The following is true regarding oocytes reserve


A. The highest oocytes reserve is at the time of parturition.
B. The development of primordial follicle disappear for sometime after parturition.
C. At menarche there are 6 to 7 million oocytes.
D. Pregnancy will influence the decline of oocytes reserve.
E. Stimulation drugs inhance the decline of oocytes reserve.

38. The following is true regarding spermiogenesis (E)


A. The development of diploid to haploid
B. Process of spermatogonia to primary spermatocyte.
C. Process of primary oocyte to secondary spermatocyte.
D. Process of secondary spermatocyte to spermatide.
E. Maturation process of spematide to spermatozoa.

39. The following is true regarding FSH in early cycle (C)


A. Stimulates IGF-1 secretion
B. Stimulates IGF-2 secretion
C. Stimulates inhibin secretion
D. Inhibits activin secretion
E. Stimulate androgen synthesis.

40. The following is true regarding follicle atresia in early cycle (A)
A. Androgenic micro environment is more dominant.
B. Has more FSH receptors in the granulosa cells.
C. The role of intra follicular activin is more dominant.
D. It is not found in normal ovulatory cycle.
E. It can disturb menstrual cycle.

41. In the normal menstrual cycle corpus luteum secretes the following hormones,
except
A. Estriol
B. Progesterone
C. Inhibin-A
D. Androgen
E. Estradiol

42. The following substances influence the amount of blood during menstruation,
except :
A. Oxytocin
B. Matrix metallo proteinase (MMP)
C. Tissue inhibitor metalloproteinase (TIMP)
D. Transforming growth factor (TGF)-β
E. Plasminogen activator inhibitor-1

43. In the normal menstrual cycle, bleeding stop due to the following, except
A. The total shedding of endometrium
B. Blood clotting factors
C. Uterine contraction
D. Vascular vasoconstriction.
E. Prostaglandins

44. The following hormones have intracellular receptor, except (C)


A. Estradiol
B. Progesterone
C. FSH
D. Testosterone
E. Androstenedion.

45. The following is true regarding hCG (A)


A. α- subunit hCG is the same with LH
B. Secreted by corpus luteum
C. It’s secretion is initiated at the time of fertilization.
D. The peak concentration is in term pregnancy
E. In ectopic pregnancy it’s concentration is higher than in normal pregnancy.

46. A patient with PCOS often have an increase in insuline resistence. This will result
in an increase in
A. FSH.
B. Free estrogen level
C. Free testosterone level
D. Hepatic production of SHBG.
E. Progestogen levels.

47. Which of the following substances is the precusor to protaglandins (PGs) (A)
A. Arachidonic acid
B. Isobutyric acid
C. Isoleucocine
D. Linoleic acid
E. Phospholipase A

48. Given that prostaglandins appear to be involved in preterm labor, which of the
following medications might provide some help in stopping preterm labor (B)
A. ACTH
B. Indomethacin
C. Progesterone
D. Prolactin-inhibiting-factor.
E. Thyroid hormone.

49. Menarche usually occurs between the ages of (B)


A. 8 and 10 years
B. 11 and 13 years
C. 14 and 16 years
D. 17 and 18 years
E. Any time.
50. In addition of ethynil group at the 17C position of estradiol was critical in the
development of the oral contraceptive pill because it (E)
A. decreases biological activity
B. Increases androgenic activity
C. Increases hepatic degradation
D. Increases SHBG affinity
E. Maintain biological activity after oral absorbsion
A. Select the one that is the best of the items
1. Prof. Dr. Samsulhadi SpOG(K) (Pembuat soal no.1 – 20)

19. The following statements regarding prolactin is true : (D)


A. It’s secreted by nucleus arcuatus
B. It stimulates oxytocn secretion
C. It stimulates GnRH secretion
D. The secretion is stimulated by TRH
E. It stimulates Norephineprin secretion

20. The following statements is true regarding the short loop feed back of GnRH.
(E)
A. Sex steroid relationship to pituitary.
B. Conection of GnRH with it’s own secretion.
C. Conection of hypothalamus to pituitary
D. With the down regulation mechanism
E. Through feed back of hypophyseal portal system.

21. The following is true regarding GnRH: (B)


A. It’s a steroid hormone
B. It has membrane receptor
C. It enters posterior lobe of pituitary
D. It directly enters from tractus tubero infundibular.
E. The secretion is stimulated by dopamine

22. The following is true regarding GnRH (A)


A. The secretion is stimulated by norephineprin
B. Hypothyroid will stimulate it’s secretion.
C. Longterm and high dose of GnRH will increase multiple ovulation.
D. Pineal gland hormone stimulate it’s secretion
E. Oxytocin will increase it’s degradation.

23. The following is true regarding the secretion of GnRH (C)


A. The amplitudo is higher in early cycle than in the luteal phase.
B. In early cycle the amplitudo is higher than in the midcycle.
C. In midcycle, the frequency of secretion is increase
D. In midcycle the amplitudo is higher than in the luteal phase.
E. In luteal phase the frequency is higher than in the early cycle.

24. The following is true regarding early relationship of hypothalamus-pituitary-


ovarian axis C)
A. The first initiation is prior to puberty
B. Initiated with FSH increase.
C. At puberty, central hypersensitivity to estrogen is decline.
D. Soon will lead to regular ovulation.
E. Resulting in the first folliculogenesis.

25. The following is true regarding oocytes reserve (B)


A. The highest oocytes reserve is at the time of parturition.
B. The development of primordial follicle disappear for sometime
after parturition.
C. At menarche there are 6 to 7 million oocytes.
D. Pregnancy will influence the decline of oocytes reserve.
E. Stimulation drugs inhance the decline of oocytes reserve.

26. The following is true regarding oogenesis (meiosis) ( D).


A. Oogonia has haploid chromosom
B. The first polar body (polar body I) disconnected at the early stage
of meiosis I
C. Polar body II disconnected at the time of ovulation
D. Meiosis disappear during follicular phase
E. The final result is 4 oocytes

27. The following is true regarding spermiogenesis (E)


A. The development of diploid to haploid
B. Process of spermatogonia to primary spermatocyte.
C. Process of primary oocyte to secondary spermatocyte.
D. Process of secondary spermatocyte to spermatide.
E. Maturation process of spematide to spermatozoa.

28. The following is true regarding the situation of early cycle (E)
A. The LH receptor is in granulosa cells
B. FSH stimulates the expression of of LH receptor.
C. Estradiol and FSH stimulate LH receptor.
D. Progesteron stimulates the secretion center of gonadotropin
E. Estradiol inhibits the secretion center of gonadotropin

29. The following is true regarding FSH in early cycle (C)


A. Stimulates IGF-1 secretion
B. Stimulates IGF-2 secretion
C. Stimulates inhibin secretion
D. Inhibits activin secretion
E. Stimulate androgen synthesis.

30. The following is true regarding follicle atresia in early cycle (A)
A. Androgenic micro environment is more dominant.
B. Has more FSH receptors in the granulosa cells.
C. The role of intra follicular activin is more dominant.
D. It is not found in normal ovulatory cycle.
E. It can disturb menstrual cycle.

31. The dominant follicle is primarily supported by the concentration of ( D)


A. Progesterone
B. IGF-1
C. IGF-2
D. Inhibin-B
E. Activin

32. The dominant follicle is formed on day ..... of the menstrual cycle ( C)
A. 12 – 14
B. 9 – 11
C. 5 – 7
D. 2 – 4
E. 1 – 2

33. In the normal menstrual cycle corpus luteum secretes the following hormones,
except (A)
A. Estriol
B. Progesterone
C. Inhibin-A
D. Androgen
E. Estradiol

34. At the time of LH surge, the following statements are true, except :( A)
A. Progesterone secretion is sharply increase.
B. LH surge happened.
C. Granulosa cells secrete prostaglandins
D. High concentration of estradiol serum
E. Oocyte maturation inhibitor (OMI) destroyed.

35. The following substances influence the amount of blood during menstruation,
except : (A)
A. Oxytocin
B. Matrix metallo proteinase (MMP)
C. Tissue inhibitor metalloproteinase (TIMP)
D. Transforming growth factor (TGF)-β
E. Plasminogen activator inhibitor-1

36. In the normal menstrual cycle, bleeding stop due to the following, except
( C)
A. The total shedding of endometrium
B. Blood clotting factors
C. Uterine contraction
D. Vascular vasoconstriction.
E. Prostaglandins

37. The following hormones have intracellular receptor, except (C)


A. Estradiol
B. Progesterone
C. FSH
D. Testosterone
E. Androstenedion.

38. The following is true regarding hCG (A)


A. α- subunit hCG is the same with LH
B. Secreted by corpus luteum
C. It’s secretion is initiated at the time of fertilization.
D. The peak concentration is in term pregnancy
E. In ectopic pregnancy it’s concentration is higher than in normal
pregnancy.

2.Prof..Dr.Djaswadi Dasuki , PhD, SpOG(K) (Pembuat soal no.21- 40)

39. Gonadotropin releasing hormone (GnRH) stimulates the release of (C):


A. Adrenocorticotropic hormone (ACTH)
B. Growth hormone (GH)
C. Luteinizing hormone (LH)
D. Opiate peptide
E. Thyroid-stimulating hormone.

40. Since the work up of an elevated prolactin level can be expensive, it is


appropriate to draw prolactin levels: (C)
A. Decrease shortly after sleep.
B. Increase after ingesting high-carbohydrate meals.
C. Increase during stress
D. Decrease during surgery
E. Decrease after excercise.
41. Low-density lipoprotein (LDL) cholesterol serves as the principal substrate for
steroidogenesis. Which of the following statements regarding circulating LDL
correct ? (C)
A. LDL is formed after addition of triglyceride to very low-density
lipoprotein (VLDL).
B. LDL levels are negatively correlated with cardiovascular disease.
C. LDL is the major carrier of cholesterol in the plasma.
D. LDL enters the cells by passive diffusion.
E. LDL facilitates the transport of polar lipids in the blood plasma.

42. The progressive sequence (with steps omitted) in the metabolism of steroid
hormone is (E)
A. Cholesterol-estriol-testosterone-pregnennolone.
B. Cholesterol-pregnenolone- cortisol-estradiol.
C. Cholesterol-pregnenolone-estrone-androstenedione.
D. Cholesterol-androstenedione- pregnenolone- estrone.
E. Cholesterol-pregnenolone-androstenedione-estrone.

43. In addition of ethinyl group at the 17C position of estradiol was critical in the
development of the oral contraceptive pill because it (E)
A. Decreases biological activity
B. Increases androgenic activity
C. Increases hepatic degradation
D. Increases SHBG affinity
E. Maitains biological activity after oral absorbtion.

44. A patient with PCOS often have an increase in insuline resistence. This will
result in an increase in (C)
A. FSH.
B. Free estrogen level
C. Free testosterone level
D. Hepatic production of SHBG.
E. Progestogen levels.

45. Which of the following substances is the precusor to protaglandins (PGs) (A)
A. Arachidonic acid
B. Isobutyric acid
C. Isoleucocine
D. Linoleic acid
E. Phospholipase A

46. Given that prostaglandins appear to be involved in preterm labor, which of the
following medications might provide some help in stopping preterm labor (B)
A. ACTH
B. Indomethacin
C. Progesterone
D. Prolactin-inhibiting-factor.
E. Thyroid hormone.

47. Which of the following embryonic tissues contributes to the adult fallopian
tubes? (D)
A. Coelemic epithelium
B. Mesenchyme
C. Mesonephric duct
D. Paramesonephric duct
E. Urogenital ridge.

48. The most common ovarian lession associated with excessive estrogen
stimulation infants is a (D)
A. Granulosa cell tumor
B. Leiomyoma
C. Serious cysadenoma
D. Single large follicular cyst
E. Theca cell tumor.

49. Menarche usually occurs between the ages of (B)


A. 8 and 10 years
B. 11 and 13 years
C. 14 and 16 years
D. 17 and 18 years
E. Any time.

50. Which of the following pubertal events is not mediated by gonadal estrogen
production (C).
A. Breast development
B. Menstruation
C. Pubic hair growth
D. Skeletal growth
E. Vaginal cornification

51. Which of the following statements best discribes the role of FSH in
menstruation? (E)
A. FSH stimulates follicle growth only in the early preanthral stage.
B. FSH increases its own receptor number on theca cells.
C. FSH induces theca cell aromatase.
D. FSH stimulates granulosa cell androgen production
E. FSH induces granulosa cell LH receptors within the dominat
follicle.

52. The midcycle LH surge (E)


A. Enhances thecal cell androgen production
B. Luteinizes granulosa cells.
C. Initiates resumption of meiosis
D. Facilitates oocyte expulsion
E. All of the above.

53. Which of the following gametes is released from the graafian follicle during
ovulation ? (D)
A. Primary oocyte
B. Primary oocyte and first polar body
C. Secondary oocyte
D. Secondary oocyte and first polar body
E. Secondary oocyte and second polar body.

54. Which of the following hormone(s) is/are produced by the corpus luteum ? (D)
A. Progestogen only
B. Progestogen and estrogen only
C. Progestogen, estrogen, and inhibin only
D. Progestogen, estrogen, inhibin and relaxin only.
E. Progestogen, estrogen, inhibin, relaxin and contractin.

55. During the evaluation for infertility, a woman may have an endomertial biopsy
to evaluate the quality of her ovulation since the development of the corpus
luteum is most closely associated with (D)
A. Fertilization of an ovum
B. Follicular phase of the endometrium
C. Proliferative phase of the endometrium
D. Secretory phase of the endometrium
E. Sheding phase of the endometrium.

56. The post coital test used in an infertility evaluation assesses the cervical mucus
for ferning. The presence of ferning depends on which of the following
hormones ? (A)
A. Estrogen
B. Estrogen and progesterone
C. hCG
D. LH
E. Progesteron.

57. Spinnbarkeit describes the (C)


A. Amount of cervical mucus
B. Clarity of cervical mucus
C. Elasticity of cervical mucus
D. Ferning of cervical mucus
E. Viscocity of crvical mucus.

58. Menstrual flow is associated with the (E)


A. Prolonged maintenence of estrogen
B. Prolonged maintenence of progesterone
C. Withdrawal of FSH
D. Withdrawal of LH

E. Withdrawal of progesterone.

3. Dr.Zain Alkaff SpOG(K) (Pembuat soal no.41 – 60 )

59. Even after menopause most women have circulating estrogen. It mainly
originates from the aromatization of (C)
A. Androstenedione to estrone by ovarian granulosa cells
B. Androstenedione to estrone by ovarian theca cells
C. Androstenedione to estrone by adipose tissue
D. Testosterone to estradiol by adipose tissue tissue
E. Estradiol to estrone by adipose tissue.

60. A. 50-year-old woman presents to her health care provider complaining of hot
flaushes. Holt flushes entail (A)
A. Peripheral redistribution of blood flow leading to sweating and
elevated heart rate.
B. Peripheral vasodilatation reflecting an increase in core body
temperature.
C. Subjective symptoms and always accompanying objective signs of
vasomotor instability.
D. Peripheral vasodilatation resulting from a direct LH action on
sympathetic neurons
E. An avarage duration of about 30 minutes.
61. With the osteoporosis the accelerated bone loss occurs 1 to 8 years after
menopause (B).
A. Causes an elevation in circulating parathyroid hormone levels
B. Causes increased urinary loss of phosphorus and hydroxyproline
C. Does not influence trabecular bone
D. Primarily affects cortical bone.
E. All of above.

62. Cardioprotective actions of estrogen may include (B)


A. Depression of HDL levels.
B. Dilatation of coronary vesels
C. Elevation of LDL levels
D. Production of thromboxane
E. Widening of pulse pressure.

63. Which of the following substances stimulates GnRH secretion? (D)


A. Beta-endorphin
B. Dopamine
C. Dynorphin
D. Norepinephrine
E. Serotonin

64. GnRH-associated peptide (GAP) is a/an. (A)


A. Stimulator of gonadotropins and an inhibitor of prolactin
B. Inhibitor of gonadotropins and a stimulator of prolactin
C. Stimulator of gonadotropins and a stimulator of prolactin
D. Inhibitor of gonadotropins and an inhibitor of prolactin
E. Stimulator of gonadotropins and has no effect on prolactin

65. Tyrosine is an essential amino acid for the biosynthesis of (A)


A. Dopamine
B. Dynorphin
C. GnRH
D. Prostaglandins
E. Vasopressin

66. A patient presents with amenorrhoea and galactorrhoea. Her prolactin levels
are elevated. She is not and never has been pregnant. In additional to
evaluating her for a prolactinoma, one also needs to evaluate for other causes
that would increase prolactin such as elevated. (E)
A. Corticotrophin-releasing hormone
B. Dopamine
C. Gamma-aminobutyric acid (GABA)
D. Histamine type II receptor activation
E. Thyrotropin-releasing hormone (TRH)

67. During which of the following conditions would the serum prolactin level be
greatest? (C)
A. Menopause
B. Ovulation
C. Parturition
D. Sleep
E. Rest

68. Which of the following statements regarding GnRH-stimulated LH secretion is


accurate? (C)
A. It is enchanced by gonadotrope exposure to progesterone
B. It is increased by gonadotrope exposure to testosterone
C. It is enchanced by gonadotrope exposure to estrogen
D. It is enchanced by gonadotrope exposure to continous GnRH
E. It is associated with steady LH release

69. Steroid hormones are classified as: (C)


A. Amino acids
B. Phospholipids
C. Lipids
D. Glycoprotein
E. None of the above

70. Adrenal corticoids belong to which of the following groups? (E)


A. Aldehydes
B. Androstanes
C. Cholesterols
D. Estranes
E. Pregnanes

71. The three principal estrogens in women in decreasing order of potency are: (B)
A. Estradiol, estriol, estrone
B. Estradiol, estrone, estriol
C. Estriol, estradiol, estrone
D. Estriol, estrone, estradiol
E. Estrone, estriol, estradiol

72. The following effects is attributed to hCG, except: (E)


A. Maintenance of corpus luteum during early pregnancy
B. Regulation of fetal adrenal androgen production
C. Regulation of fetal testicular androgen production
D. Stimulation of thyroid activity
E. Inhibition of peripheral glucose uptake

73. The following substances are placental protein hormones, except: (A)
A. Dopamine
B. Inhibin
C. GnRH
D. Human chorionic thyrotropin (hCT)
E. Thyroid-releasing hormone (TRH)

74. The following statements regarding lactation are TRUE, except: (D)
A. Estrogen stimulates mammary ductal proliferation
B. Progesterone stimulates mammary alveolar development
C. High circulating sex steroid levels during pregnancy inhibit
lactation
D. The gestational mammary gland produces a transudate containing
casein and alpha-lactalbumin
E. The postpartum decline in circulating sex steroid levels initiates
lactation

75. Management of anovulatory bleeding (dysfunctional uterine bleeding)


depends on the following conditions, except: (C)
A. Age of the patient
B. Desire for fertility
C. Size of the cervical os
D. Amount of bleeding
E. Cause of bleeding

76. Treatment of hyperprolactinemia results in the following, except: (B)


A. Elimination of galactorrhoea
B. Prevention of acne
C. Establishment of normal estrogen production
D. Treatment of prolactin-secreting pituitary adenomas
E. Induction of ovulation

77. The following statements regarding PCOS are true, except: (D)
A. Elevated androgen production decreases sex hormone-binding
globulin (SHBG) synthesis
B. Elevated androgen production increases peripheral aromatization
C. Acyclic estrogen production increases LH secretion
D. Acyclic estrogen production decreases bone mineralization
E. Acyclic estrogen production increases endometrial proliferation

78. Which of the following is manifestation of virilization? (B)


A. Intermammary hair
B. Clitoromegaly
C. Abdominal hair
D. Hair on upper legs
E. Deep voice

4.Dr.Amino Rahardjo SpOG (K) (Pembuat soal no.61 – 80 )


79. Which of the following is the principal site of production of sex hormone-
binding globulin (SHBG)? (C)
A. Kidney
B. Pituitary
C. Liver
D. Hypothalamus
E. None of above

80. Which of the following medications is most likely to cause hirsutism? (A)
A. Danazol
B. Cimetidine
C. Tetracycline
D. Α-methyldopa
E. Diazepam

81. Which of the following is the most common cause of pathologic hirsutism? (B)
A. Congenital adrenal hyperplasia
B. Polycystic ovary syndrome
C. Cushing’s syndrome
D. Adrenal tumor
E. Turner syndrome

82. Which layer of the endometrium is refractory to progesterone? (C)


A. stratum compactum
B. stratum functionalis
C. stratum basalis
D. stratum spongiosum
E. All above

83. Endometrial biopsy is indicated in all of the following situations except: (A)
A. all women over age 35
B. women over age 35 with metrorrhagia
C. menorrhagia not responding to medical therapy
D. women at high risk for endometrial adenocarcinoma with
dysfunctional uterine bleeding (DUB)
E. women taking unopposed estrogens with DUB

84. In perimenopausal women, the following is true of the follicular and luteal
phase: (D)
A. Both phases lengthens
B. The follicular phase shortens and the luteal phase lengthens
C. The luteal phase shothens and the follicular phase lengthens
D. Both phases shorthen
E. Both phases are still in normal cycle.
85. Initial workup for a patient with postmenopausal bleeding should include all of
the following except: (D)
A. Pap smear
B. Pelvic exam
C. Rectal exam
D. Office hysteroscopy
E. Endometrial sampling

86. Adenomyosis is defined as: (B)


A. Adenosis of the intramural myometrium
B. Ectopic endometrial tissue extending more than 2 low power
fields (LPFs) deeper than the basalis layer into the myometrium
C. Dysfunctional uterine muscle that does not contract following
endometrial shedding
D. Endometrium growing at least 2 high power fields (HPFs) into the
intramural layer of the uterus
E. Endometrium growing less than 2 high power fields (HPFs) into
the intramural layer of the uterus

87. Which of the following drugs are useful in the treatment of menorrhagia? (D)
A. Cabergoline
B. Oil of evening primrose
C. Diazoxide
D. Naproxen
E. Diazepam.

88. Which is the following statements is not true? (D)


A. Pulsatile secretion of the GnRH controls the synthesis of FSH and
LH
B. Prostaglandin-induced constriction of the spiral arterioles causes
endometrial ischaemia
C. Progesterone causes decidualisation of the endometrium
D. Secretion of prolactin from the anterior pituitary is needed to
maintain a regular cycle
E. Folliculogenesis is under FSH control.

89. Regarding hirsutism, which one is not true (E)


A. SHBG is usually decreased
B. It may be successfully treated with the combined oral
contraceptive pill
C. In the presence of a regular menstrual cycle it is usually idiophatic
D. It’s commonly related with PCOS
E. It may be caused by diazepam

90. Regarding endometriosis: (A)


A. Asymptomatic deposits of endometriosis are found in 20% of
women of reproductive age
B. Once found it should always be treated
C. May not present with a painful nodule in a surgical scar
D. It’s always persists until the menopause.
E. It can be treated totally with GnRH analog.
91. Regarding the normal menstrual cycle: (D)
A. FSH promotes luteal phase development
B. Progestogen is produced by granulose cells.
C. Oestrogen is produced mainly by theca cells
D. Prostaglandins stimulate myometrial contraction.
E. Progesterone peaks in the follicular phase

92. Which of the following may cause pelvic pain? (B)


A. Retroverted mobile uterus
B. Ovarian hyperstimulation
C. Primary syphilis
D. Policystic ovary syndrome
E. Endometrial polyp.

93. Regarding the polycystic ovary syndrome: (E)


A. SHBG levels are increased
B. Androgens are decreased
C. LH secretion is decreased
D. Is not related with infertility
E. Menstrual irregularity is a common feature

94. Which one is not include in the side effects of treatment for endometriosis:
(D)
A. Acne
B. Hot flushes
C. Decrease in breast size
D. Endometrial hyperplasia
E. Weight gain

95. Which of the following statements is not true? (E)


A. Removal of both ovaries cures premenstrual syndrome
B. Visceral pain from the uterus, fallopian tubes and ovaries is
transmitted via the autonomic nervous system (T10-L1)
C. Cone biopsy of the cervix can cause secondary dysmenorrhoea
D. Marker of Ca-125 can be used as diagnostic tool of endometriosis.
E. Endometriosis related with cancer.

96. Which of the following is not true regarding the IUD? (C)
A. Serum copper levels are unchanged
B. The levonorgestrel-containing IUD is associated with a small
increase in the haemoglobin level

C. Pregnancy with IUD insitu should be terminated.


D. The copper-containing IUD is associated with a small decreased in
the haemoglobin level
E. The plastic frame of the levonorgestrel-containing IUD contains
barium so that it can be seen on X-ray
97. Which of the following is not caused by endometriosis? (D)
A. Haemoptysis
B. Infertility
C. Renal failure
D. Cancer
E. Dispareunia.

5. Prof.Dr.Mochamad Anwar M.Med.Sc, SpOG(K)

98. Which of the following statements regarding the POP (progestogen only pill)
are true? (E)
A. It works mainly by inhibiting ovulation
B. The dose of progestogen is higher than that in the combined pill
C. It does not appear in the breast milk
D. It does not influence motility of sperm
E. It is suitable for both diabetic and hypertensive women

99. Regarding the combined oral contraceptive (COC) pill: (B)


A. The incidence of iron deficiency anaemia is slightly increase
B. The COC is protective against both ovarian and endometrial
cancer
C. It does not carries an increased risk of venous thromboembolism
D. The incidence of the benign breast disease is increased by 30%
E. It is not suitable for young women for postponing pregnancy.

100. Which of the following is not true? (B)


A. Injectable progestogens inhibit lactation
B. Post-coital IUD insertion is effective if fitted within 7 days of
unprotected intercourse
C. The injectable progestogen depo-provera inhibits ovulation
D. Return of fertility may be delayed following removal of a
contraceptive implant
E. Implant contraceptive method is suitable for post partum women.

101. Regarding contraception and the menopause: (A)


A. The low-dose COC can be used up to the menopause in healthy
non-smoking women
B. Measurement of FSH is an accurate indicator of fertility
C. HRT can be used as contraception
D. Sterilization affects the timing of menopause
E. For pre-menopausal women, serum FSH concentration is not
considered for taking COC.
102. Regarding the post-coital test: (D)
A. It must be done on day 21 of the cycle
B. It can detect anovulation
C. It can detect tubal problems
D. It can detect azoospermia
E. It can detect acrosome reaction of sperm.

103. Which of the following provide evidence of ovulation? (B)


A. Day 14 progesterone level

F. Endometrial biopsy
G. Normal menstrual cycle.
H. Estrogen / progestogen ratio
I. LH / FSH ratio

104. Which of the following is not used to correct anovulation? (D)


A. Bromocriptine
B. Weight loss
C. HCG injection
D. Low-dose oestrogen
E. Clomiphen citrate

105. Which of the following may not cause male infertility? (D)
A. Gonorrhoea infection
B. Cystic fibrosis
C. Prolactinoma
D. Hydrocele
E. Varicocele

106. Which of the following are associated with early pregnancy loss? (E)
A. Sexual intercourse
B. Multiple partner
C. Wart virus infection
D. Pulmonal tuberculosis
E. Intra-uterine pregnancy with an intra-uterine device in situ

107. Regarding osteoporosis, which one is not true: (D)


A. Hip fracture causes more deaths than cancer of the cervix, uterus
and ovaries combined
B. Bone densitometry can be used to screen individual at risk
C. May be associated with hypogonadism
D. Fracture is more common on stratum compactum instead of
trabecular.
E. Osteocyt : osteoblasts that are trapped in cortical bone during the
remodeling process.

108. Which the following are associated with an increased risk of osteoporosis?
(A)
A. Malabsorption syndrome
B. Steroid therapy
C. Hyperparathyroidism
D. Late menopause
E. Premature ovarian failure (POF)

91. Case-1

A 32-year-old nulliparous woman has had oligomenorrhea since menarche.


During the past 5 years she has experienced slow but progressive increase in
hair on her face, back and forearms in the inter-mammary space, and on the
back of her hands. Her voice has slowly deepened and temporal balding and
clitoromegaly have developed. Which of the following is the most likely
diagnosis? (C)

A. Polycystic ovary syndrome

B. Cushing’s syndrome
C. Stromal hyperthecosis
D. Ovarian tumor
E. Asherman’s syndrome

92. Case-2.
A couple have a 2-year history of infertility. The male partner is healthy with
no past serious illnesses, and has one child from his previous marriage. The
female partner has a regular 29-day menstrual cycle and has a normal body
mass index. Her only previous pregnancy was an ectopione. Which of the
following investigations is most likely to reveal the cause of the infertility? (E)

A. Semen analysis
B. Pelvic ultrasound examination
C. Hormonal profile
D. Hysteroscopy
E. Laparoscopy

Case-4 (questions 93 – 95)

A 47-year-old patient presents wondering if her problems with mood, swings,


insomnia, and vaginal dryness represent menopause. She had a hysterectomy
10 years ago for abnormal uterine bleeding, but the uterus were not removed.
Since she cannot afford hormonal testing, a maturation index is done on her
pap smear.

93. Ideally, cytologic cells for evaluation of hormonal status should be obtained

from the (D)

A. Ectocervix
B. Endocervix
C. Labia minora
D. Lateral vaginal wall
E. Posterior vaginal fornix.

94. Ninety percent of the cells found on her pap smear have thick, rounded
cytoplasm and plump, round, vesicular nuclei with an intact chromatin patter.
The maturation index (MI) would most likely be (B)
A. 90 / 0 / 10.
B. 90 / 10 / 0
C. 10 / 0 / 90
D. 10 / 90 / 0.
E. 0 / 90 / 10

95. Based on this result, one would anticipate that if hormonal levels were

obtained they would show (C)

A. Elevated estrogen level


B. Elevated progesterone levels.
C. Low estrogen levels
D. Normal estrogen levels
E. Normal progesterone levels

Case-5

A patient presented to the emergency department with an infected


incomplete abortion. During the dilatation and curettage (D&C) excessive
bleeding developed that required vigorous curetting to control. She returns to
the physician 6 months later complicating that she has not had a menstrual
cycle. She has all the symptoms of getting ready to start a period but never
sees any bleeding.

96. This history implies that what layer of endometrium is damaged ? (A)
A. Basal zone

B. Compact zone
C. Functional zone
D. Spongy zone
E. None of above
6. Prof.Dr.Samsulhadi SpOG (K) (Pembuat soal 97 – 100 )

Case-6. (for questions 97 – 100 )

Woman of 30 years, married for 5 years and no children, seeing the doctor for
planning pregnancy. The menstruation has been lasting for 10 days with
irregular period. The body height is 155 cm, body weight 80 kg, blood pressure
140/90, pulse rate 80/minute. Vaginal ultrasound examination: the size of
uterus is normal, no gestational sac, endometrial thick 1.9 mm, enlargement of
both ovaries with > 10 follicles of 7-8 mm diameter on a necklace type, stroma is
thick. Two times sperm analysis is azoospema, FSH of husband is 6 mIU/ml.

97. Based on the Roterdam consensus 2003, for the diagnosis of PCOS on this
case, is still needed to exam : (E)
A. Serum estradiol
B.Ratio LH/FSH
C.Fasting insulin serum
D. Total testosterone serum
E.None of above.
98. The first step to overcome menstrual problems for this case should be
treated with (D)
A. GnRH agonist
B. GNRH antagonist
C. Aromatase inhibitor
D. Progestogen
E. Uterotonic

99. In the long term, this women will be in a high risk of (A)
A. NID-DM

B. Myometrial malignancy

C. Ovarial malignancy

D. Osteoporosis

E. Alzeimer.

100. ART problems might be happened in this case (D)


A. Low FSH serum concentration

B. High estradiol serum concentration

C. High androgen serum concentration

D. High LH serum concentration

E. Low progesterone serum concentration.


1. The differences between RNA and DNA are, except :

a. RNA uses pentose sugar


b. RNA is single stranted
c. RNA contain uracil
d. Sugar in RNA is ribose
e. RNA carries codon

What happen in increase Corticotrophin Releasing Hormone (CRH) in fetal development, All
statement in bellow are true; except :

A. Increase fetal cortisol is positive feedback relationship

B. Augment fetal ACTH secretion.

C. Adjunct progesterone secretion.

D. More DHEAS.

E. Increase estrogen

2. Which regard of the following is related to the first sign of pubertal development of
female :
a. Menarche
b. Menopause
c. Pubarche
d. Menstruation
e. Telarche

3. Mechanism of action for steroid hormones :

A. Rapidly transported across the cell membrane by osmose action.

B. All of which require direct interaction with DNA.

C. Via hormone-receptor complex to bind hormone-responsive element in DNA.

D. Not regulate posttranscriptional events.

E. Translation via DNA methylation.

4. This statement is not true about basic mechanisms of menstruation

a. Menstruation is preceeded by intense vasoconstriction of the spiral arterioles


b. Ischaemia induced by intense vasoconstriction of 4 to 24 hours will be
augmented by increased release of free oxygen radicals produced by tissue
(endometrial) desquamation
c. Platelet fibrin plug formation is grossly deficient in endometrium
d. Ruptured spiral arterioles do not form platelet fibrin thrombi in the early
phases of menstruation
e. The rapid formation of fibrin plugs and their degradation play a part of the
control of menstrual bleeding

5.The purpose of the progestational challenge test is to asses the level of endogenous :
A. Testosterone
B. Progesterone
C. Estrogen
D. Prolactin
E. TSH

6. Which of the following sequences best describes estrogen action?

a. cell membrane diffusion, steroidreceptor,-DNA complex formation, transcription,


translation

b.cell membrane receptor activation, steroidreceotor DNA complex formation,


translation,transcription, transcription

c. cell membrane diffusion' steroidreceptor-DNA complex formation, translation,


transcription

d.cell membrane diffusion, adenylatecyclase activation, cAMP production, protein


phosphorylation

e. cell membrane receptor activation, adenylatecyclase activation, c AMP production,


protein phosphorilation..

7.Amenorhea should be evaluated in any patient without a periode who has an absence of
growth or development of secondary sexual characteristic by age
A. 8
B. 10
C. 12
D. 14
E. 16

8.Sperm capacitation refers to a process by which spermatozoa become capable of:

a. stimulating meiosis of ovum

b. dispersing the zona radiata

c. penetrating the cervical mucus

d. producing acrosomal enzymes

e. fertilizing the ovum


9.Gonadotropin releasing hormone (GnRH) stimulates the release of:

a. Adrenocorticotropic hormone (ACTH)


b. Growth hormone (GH)
c. Luteinizing hormone (LH)
d. Opiate peptide
e. Thyroid-stimulating hormone (TSH)

10. This statement is not true about basic mechanisms of menstruation

a. Menstruation is preceeded by intense vasoconstriction of the spiral arterioles

b. Ischaemia induced by intense vasoconstriction of 4 to 24 hours will be augmented by


increased release of free oxygen radicals produced by tissue (endometrial)
desquamation

c. Platelet fibrin plug formation is grossly deficient in endometrium

d. Ruptured spiral arterioles do not form platelet fibrin thrombi in the early phases of
menstruation

e. The rapid formation of fibrin plugs and their degradation play a part of the control of
menstrual bleeding

11. Which of the following refers to WHO consensus 1999 about normosperm :

a. consentration less than 20 million/ml


b. volume at least 2 ml
c. grade A at least 50%
d. grade A + B more than 2%
e. leucocyte less than 2/ml

12.Regulation of fetal adrenal gland in relation with labor process, in late gestation :

A. The mother prevent high cortisol by converting to cortisone.

B. Increasing estogen maternal.

C. Increasing 11β- hydroxysteroid dehydrogenase.

D. A, B and C true.

E. A and C true.

13.29-year-old primigravida who received no prenatal care has marked vaginal bleeding
after the onset of labor at 38 weeks gestation. Cesarean section is performed and a
lacerated low-lying placenta is removed. She remains hypotensive for 6 hours and
requires transfusion of 12 packed RBC units. Postpartum, she becomes unable to
breast-feed the infant. She does not have a resumption of normal menstrual cycles. She
becomes more sluggish and tired. Laboratory findings include hyponatremia,
hyperkalemia, and hypoglycemia. Which of the following pathologic lesions is she most
likely to have had following delivery?
A  Bilateral adrenal hemorrhage
B   Pituitary necrosis
C   Subacute thyroiditis
D   Metastatic choriocarcinoma
E   Insulitis

14. A patient presents with amenorrhea and galactorrhea. Her prolactin level are elevated.
She is not and never has been pregnant. In addition to evaluating her for a
prolactinoma, one also needs to evaluate for other causes that would increase prolactin
such as elevated.

a. dopamine

b. gamma-aminobutyric acid

c. hystamin type II receptor activation

d. Thyrotropin releasing hormone

e. corticotrophin releasing hormone

15. The initial step in the workup of amenorheic patient after excluding pregnancy begin
with measurement of :
A. TSH
B. LH
C. AMH
D. FSH
E. GnRH

16. Which of the following sequences best describes estrogen action?

a. cell membrane diffusion, steroidreceptor,-DNA complex formation, transcription,


translation

b. cell membrane receptor activation, steroidreceotor DNA complex formation,


translation, transcription, transcription

c. cell membrane diffusion' steroidreceptor-DNA complex formation, translation,


transcription

d. cell membrane diffusion, adenylatecyclase activation, cAMP production, protein


phosphorylation

e. cell membrane receptor activation, adenylatecyclase activation, c AMP production,


protein phosphorilation..

17. Physical examination of amenorhea :


A. Body mass index
B. Breast development
C. The presence of pubic hair growth
D. Evaluation of the genital outflow and uterus
E. If all are correct

18. Which of the following causes of delayed puberty accompanies elevated circulating
gonadotropin levels?

a. Kallman's syndrome

b. Hypothalamic tumors

c. gonadaldysgenesis

d. malnutrition

e. chronic illness

19. Which of the following sequences best describes estrogen action?

a. cell membrane diffusion, steroidreceptor,-DNA complex formation, transcription,


translation

b. cell membrane receptor activation, steroidreceotor DNA complex formation,


translation, transcription, transcription

c. cell membrane diffusion' steroidreceptor-DNA complex formation, translation,


transcription

d. cell membrane diffusion, adenylatecyclase activation, cAMP production, protein


phosphorylation

e. cell membrane receptor activation, adenylatecyclase activation, c AMP production,


protein phosphorilation..

20.What happen in increase Corticotrophin Releasing Hormone (CRH) in fetal development,


All statement in bellow are true; except :

A. Increase fetal cortisol is positive feedback relationship

B. Augment fetal ACTH secretion.

C. Adjunct progesterone secretion.

D. More DHEAS.

E. Increase estrogen
Compared to users of combination oral contraceptive, users of progestin
only are likely to experience?
A. Intrauterine pregnancy
B. Irregular vaginal bleeding
C. Gonadotropin suppression
D. Ectopic pregnancies
E. Mood swings

02. The primary mechanism by which oral contraceptive prevent pregnancy?


A. Inhibiting serum FSH levels
B. Inhibiting serum LH levels
C. Inducing endometrial atrophy
D. Inducing lymphatic endometritis
E. Increasing cervical mucus viscosity

03. The primary mechanism of IUD is


A. Creating chronic endometritis
B. Preventing fertilization
C. Inhibiting ovulation
D. Altering tubal motility
E. Destroying sperm

04. Which of the following surgical approaches for sterilization procedure is


associated with the highest failure rate?
A. Postpartum mini laparotomy
B. Interval mini laparotomy
C. Laparoscopy
D. Hysteroscopy
E. Vaginal colpotomy

05. Reducing the estrogen content of oral contraceptive results in an


increase in the rate of?
A. Pregnancy
B. Breakthrough bleeding
C. Thromboembolic complication
D. Insulin resistance
E. Premenstrual syndrome

06. The administration of RU 486 result in


A. Abortion when givinen in early pregnancy
B. Delayed menses when given during midluteal phase
C. Menses when given during follicular phase
D. Resistance to prostaglandin inhibitors
E. Induction of progesterone receptors in endometrium

07. A 31 year old infertility patient with regular ovulatory menstrual sycles
has begun theraphy with clomiphene citrate. Before she starts theraphy,
what information should you provide her regarding the medication?
A. The timing of ovulation is increased by a a week
B. Approximately 40% of patient will respond to clomiphene citrate with
increased endometrial thickness
C. The risk of multiple gestation is 25%
D. CC improves the fecundity rate principally through its effect on the
endometrial lining
E. Risk and side effects of CC include nausea, hot flushes, weight gain
and mood swings

08. The initial treatment of choice in a patient with hypogonadotric


hypogonadism when ovulation is desired is
A. Low dose estrogen theraphy
B. Human menopausal gonadotropin (hMG) theraphy
C. Bromocriptine mesylate
D. Clomiphene citrate
E. Cyclic progesterone

09. Which of the following statements best describes estrogen action on


cervical mucus?
A. It decreases the water content of cervical mucus
B. It decreases the palm-leaf crystallization pattern of mucus upon
drying
C. It decreases formation of glycoprotein channels favoring sperm
penetration
D. It increases cervical mucus stretchability (spinkerbait)
E. It increases the amount of potassium chloride in the cervical mucus

10. The highest rate complication is associated with second-trimester


pregnancy termination using?
A. Intravenous oxytocin
B. Intravenous prostaglandin
C. Intravaginal prostaglandin
D. Intramuscular prostaglandin
E. Dilation & evacuation

11. Which of the following statements regarding basal body temperature is


true?
A. An oral temperature is taken prior to bedtime
B. a rise of 0,2°F between 2 consecutive days reflects ovulation
C. a biphasic temperature shift reflects estrogen action on the
hypothalamus
D. absence of biphasic temperature shift suggest pregnancy
E. none of the above
12. Which of the following is the best method to time intercourse for
procreative means?
A. Thermogenic shift in basal body temperature
B. Urinary luteinizing hormone (LH) kit testing
C. Serum progesterone level
D. Profuse, thin, acellular cervical mucus
E. Mittelschemerz

13. IUD s are associated with an increase in


A. Salpingitis in the first few weeks of use
B. Salpingitis in long term users
C. The number of ectopic pregnancy
D. Dysmenorrhea in multiparous patients
E. The rate of PID
14. Which of the following statements regarding the postcoital test is true?
A. It predicts whether pregnancy can occur
B. It is performed 1 to 2 days after ovulation
C. It correlates the number of sperm in the cervical mucus with the
pregnancy rate
D. It examines the ability of sperm to reach and survive in the mucus
E. It is performed within 1 hour of coitus

15. Besides infertility, the most common symptoms of luteal phase defects is
A. Vaginal dryness
B. Spontaneous miscarriage
C. Tubal occlusion
D. Breast tenderness
E. Ovarian enlargement

16. Which of the following causes for infertility may be treatable by assisted
reproductive technology?
A. Fallopian tube obstruction
B. Low sperm count
C. Cervical mucus abnormalities
D. Unexplained infertility
E. All of the above

17. Infertility from endometriosis may be due to which four of the following
abnormalitie, except?
A. Pelvic adhesions
B. altered fallopian tube motility
C. sperm phagocytosis by peritoneal macrophages
D. defective embryo implantation
E. pituitary failure

18. Which of the following hormones decreases after first trimester


pregnancy?
A. Progesterone
B. Prolactin
C. hcg
D. Human placental lactogen
E. Estriol

19. A 22 years old woman with amenorrhea of 6 weeks duration undergoes


surgery for acute appendicitis. At the time of surgery, a 3 cm semisolid
left ovarian cyst is discovered. It is vascular and appears to contain a
blood filled central cavity. A serum pregnancy test is positive. Which of
the following procedure should be done?
A. Ovarian cystectomy
B. Ovarian wedge resection
C. Oophorectomy
D. Salphingo-oophorectomy
E. None of the above
20. A 16 year old girl has not experienced menarche. Examination shows
absence of breast development and small but otherwise normal female
pelvic organs. Which of the following diagnostic tests is most useful in
determining the etiology of the amenorrhea?
A. FSH
B. Serum estradiol
C. Serum testosterone
D. MRI of the head
E. Ovarian biopsy

21. A 28 year old patient complains of amenorrhea after D&C for postpartum
bleeding. The most likely diagnosis is?
A. Gonadal dysgenesis
B. Shehaan syndrome
C. Kallman syndrome
D. Mayer-Rokitansky-Kuster-Hauser syndrome
E. Asherman syndrome

22. Prolactin secreting pituitary adenomas usually?


A. Diminish in size during pregnancy
B. Increase in size over time
C. Are symptomatic during lactation
D. Impinge on the olfactory nerve
E. Respond to medical therapy

23. Management of anovulatory bleeding (dysfunctional uterine bleeding)


depends on which four following conditions, except?
A. Age of the patient
B. Desire for fertility
C. Size of cervical os
D. Amount of bleeding
E. Cause of bleeding

24. Treatment of hyperprolactinemia results in which four of the following,


except?
A. Elimination of galactorrhea
B. Prevention of acne
C. Establishment of normal estrogen production
D. Treatment of prolactin-secreting pituitary adenomas
E. Induction of ovulation

25. Hirsutism is associated with which four of following conditions?


A. PCOS
B. CAH
C. Cushing syndrome
D. Increased androgen utilization by skin
E. Hyperprolactinemia

26. Anovulatory bleeding (dysfunctional uterine bleeding) is?


A. Common in prepubertal girls
B. Common in the early teens
C. Usually suggestive of steroid-production ovarian tumors
D. Uncommon during the perimenopausal years
E. Dependent on the presence of progesterone

27. An 18 year old woman comes to your clinic with irregular cycles since
menarche and mild hirsutism. She is not interested in pregnancy or
contraception. Her serum TSH, prolactin, and DHEAS levels are normal,
with slightly elevated serum testosterone level of 80 ng/dL. Which of the
following is the most appropriate next step for this patient?
A. Oral contraceptive treatment
B. Endometrial biopsy
C. GnRH stimulation test
D. Clomiphene citrate
E. Bromocriptine

28. Which contraceptive method has the lowest pregnancy rate in 100
woman using the method for 1 year?
A. IUD
B. Long acting progestins
C. Diaphragm
D. Oral contraceptives
E. Spermicidal cream

29. Spermicides destroy spermatozoa primarily by?


A. Activating acrosomal enzymes
B. Disrupting cell membranes
C. Inhibiting glucose transport
D. Altering vaginal enzymes
E. Increasing vaginal pH

30. In a young, obese, chronically anovulatory woman with an elevated


LH:FSH ratio and polycystic appearing ovaries, which of the following if
the preferred initial method of ovulation induction?
A. Metformin
B. Human menopausal gonadotropins
C. Pulsatile GnRH
D. Clomiphene citrate
E. Bromocriptine mesylate
A patient with PCOS often have an increase in insuline resistence. This will result
in an increase in
A. FSH.
B. Free estrogen level
C. Free testosterone level
D. Hepatic production of SHBG.
E. Progestogen levels.

2. Which of the following substances is the precusor to protaglandins (PGs)


A. Arachidonic acid
B. Isobutyric acid
C. Isoleucocine
D. Linoleic acid
E. Phospholipase A

3. Given that prostaglandins appear to be involved in preterm labor, which of


the following medications might provide some help in stopping preterm
labor
A. ACTH
B. Indomethacin
C. Progesterone
D. Prolactin-inhibiting-factor.
E. Thyroid hormone.

4. During the evaluation for infertility, a woman may have an endomertial


biopsy to evaluate the quality of her ovulation since the development of the
corpus luteum is most closely associated with
A. Fertilization of an ovum
B. Follicular phase of the endometrium
C. Proliferative phase of the endometrium
D. Secretory phase of the endometrium
E. Sheding phase of the endometrium.

5. The post coital test used in an infertility evaluation assesses the cervical
mucus for ferning. The presence of ferning depends on which of the
following hormones ?
A. Estrogen
B. Estrogen and progesterone
C. Hcg
D. LH
E. Progesteron.

6. Management of anovulatory bleeding (dysfunctional uterine bleeding)


depends on the following conditions, except:
A. Age of the patient
B. Desire for fertility
C. Size of the cervical os
D. Amount of bleeding
E. Cause of bleeding

7. Treatment of hyperprolactinemia results in the following, except:


A. Elimination of galactorrhoea
B. Prevention of acne
C. Establishment of normal estrogen production
D. Treatment of prolactin-secreting pituitary adenomas
E. Induction of ovulation

8. The following statements regarding PCOS are true, except:


A. Elevated androgen production decreases sex hormone-binding
globulin (SHBG) synthesis
B. Elevated androgen production increases peripheral aromatization
C. Acyclic estrogen production increases LH secretion
D. Acyclic estrogen production decreases bone mineralization
E. Acyclic estrogen production increases endometrial proliferation

9. Which of the following is manifestation of virilization?


A. Intermammary hair
B. Clitoromegaly
C. Abdominal hair
D. Hair on upper legs
E. Deep voice

10. Endometrial biopsy is indicated in all of the following situations except:


A. all women over age 35
B. women over age 35 with metrorrhagia
C. menorrhagia not responding to medical therapy
D. women at high risk for endometrial adenocarcinoma with
dysfunctional uterine bleeding (DUB)
E. women taking unopposed estrogens with DUB

11. In perimenopausal women, the following is true of the follicular and luteal
phase:
A. Both phases lengthens
B. The follicular phase shortens and the luteal phase lengthens
C. The luteal phase shothens and the follicular phase lengthens
D. Both phases shorthen
E. Both phases are still in normal cycle.
12. Initial workup for a patient with postmenopausal bleeding should include
all of the following except:
A. Pap smear
B. Pelvic exam
C. Rectal exam
D. Office hysteroscopy
E. Endometrial sampling
13. Adenomyosis is defined as: (B)
A. Adenosis of the intramural myometrium
B. Ectopic endometrial tissue extending more than 2 low power fields
(LPFs) deeper than the basalis layer into the myometrium
C. Dysfunctional uterine muscle that does not contract following
endometrial shedding
D. Endometrium growing at least 2 high power fields (HPFs) into the
intramural layer of the uterus
E. Endometrium growing less than 2 high power fields (HPFs) into the
intramural layer of the uterus

14. Which of the following drugs are useful in the treatment of menorrhagia?
A. Cabergoline
B. Oil of evening primrose
C. Diazoxide
D. Naproxen
E. Diazepam.

15. Which is the following statements is not true?


A. Pulsatile secretion of the GnRH controls the synthesis of FSH and LH
B. Prostaglandin-induced constriction of the spiral arterioles causes
endometrial ischaemia
C. Progesterone causes decidualisation of the endometrium
D. Secretion of prolactin from the anterior pituitary is needed to maintain
a regular cycle
E. Folliculogenesis is under FSH control.

16. Regarding hirsutism, which one is not true


A. SHBG is usually decreased
B. It may be successfully treated with the combined oral contraceptive pill
C. In the presence of a regular menstrual cycle it is usually idiophatic
D. It’s commonly related with PCOS
E. It may be caused by diazepam

17. Regarding endometriosis:


A. Asymptomatic deposits of endometriosis are found in 20% of women
of reproductive age
B. Once found it should always be treated
C. May not present with a painful nodule in a surgical scar
D. It’s always persists until the menopause.
E. It can be treated totally with GnRH analog.

18. Regarding the normal menstrual cycle:


A. FSH promotes luteal phase development
B. Progestogen is produced by granulose cells.
C. Oestrogen is produced mainly by theca cells
D. Prostaglandins stimulate myometrial contraction.
E. Progesterone peaks in the follicular phase
19. Which of the following may cause pelvic pain?
A. Retroverted mobile uterus
B. Ovarian hyperstimulation
C. Primary syphilis
D. Policystic ovary syndrome
E. Endometrial polyp.
20. Regarding the polycystic ovary syndrome:

A. SHBG levels are increased

B. Androgens are decreased

C. LH secretion is decreased

D. Is not related with infertility

E. Menstrual irregularity is a common feature

21. Which one is notinclude in the side effects of treatment for endometriosis except :

A. Acne

B. Hot flushes

C. Decrease in breast size

D. Endometrial hyperplasia

E. Weight gain

22. Which of the following statements is not true?

A. Removal of both ovaries cures premenstrual syndrome

B. Visceral pain from the uterus, fallopian tubes and ovaries is transmitted via the
autonomic nervous system (T10-L1)

C. Cone biopsy of the cervix can cause secondary dysmenorrhoea

D. Marker of Ca-125 can be used as diagnostic tool of endometriosis.

E. Endometriosis related with cancer.

23. Which of the following is not true regarding the IUD?

A. Serum copper levels are unchanged

B. The levonorgestrel-containing IUD is associated with a small increase in the


haemoglobin level

C. Pregnancy with IUD insitu should be terminated.


D. The copper-containing IUD is associated with a small decreased in the
haemoglobin level

E. The plastic frame of the levonorgestrel-containing IUD contains barium so that it


can be seen on X-ray

24. Which of the following is not caused by endometriosis?

A. Haemoptysis

B. Infertility

C. Renal failure

D. Cancer

E. Dispareunia.

25. Which of the following statements regarding the POP (progestogen only pill) are true?

A. It works mainly by inhibiting ovulation

B. The dose of progestogen is higher than that in the combined pill

C. It does not appear in the breast milk

D. It does not influence motility of sperm

E. It is suitable for both diabetic and hypertensive women

26. Regarding the combined oral contraceptive (COC) pill:

A. The incidence of iron deficiency anaemia is slightly increase

B. The COC is protective against both ovarian and endometrial cancer

C. It does not carries an increased risk of venous thromboembolism

D. The incidence of the benign breast disease is increased by 30%

E. It is not suitable for young women for postponing pregnancy.

27. Which of the following is not true?

A. Injectable progestogens inhibit lactation

B. Post-coital IUD insertion is effective if fitted within 7 days of unprotected


intercourse

C. The injectable progestogen depo-provera inhibits ovulation

D. Return of fertility may be delayed following removal of a contraceptive implant

E. Implant contraceptive method is suitable for post partum women.


28. Regarding contraception and the menopause:

A. The low-dose COC can be used up to the menopause in healthy non-smoking


women

B. Measurement of FSH is an accurate indicator of fertility

C. HRT can be used as contraception

D. Sterilization affects the timing of menopause

E. For pre-menopausal women, serum FSH concentration is not considered for


taking COC.

29. Regarding the post-coital test:

A. It must be done on day 21 of the cycle

B. It can detect anovulation

C. It can detect tubal problems

D. It can detect azoospermia

E. It can detect acrosome reaction of sperm.

30. Which of the following provide evidence of ovulation?

A. Day 14 progesterone level

B. Endometrial biopsy

C. Normal menstrual cycle.

D. Estrogen / progestogen ratio

E. LH / FSH ratio

31. Which of the following is not used to correct anovulation?

A. Bromocriptine

B. Weight loss

C. HCG injection

D. Low-dose oestrogen

E. Clomiphen citrate

32. Which of the following may not cause male infertility?


A. Gonorrhoea infection

B. Cystic fibrosis

C. Prolactinoma

D. Hydrocele

E. Varicocele

33. Which of the following are associated with early pregnancy loss?

A. Sexual intercourse

B. Multiple partner

C. Wart virus infection

D. Pulmonal tuberculosis

E. Intra-uterine pregnancy with an intra-uterine device in situ

34. Regarding osteoporosis, which one is not true:

A. Hip fracture causes more deaths than cancer of the cervix, uterus and ovaries
combined

B. Bone densitometry can be used to screen individual at risk

C. May be associated with hypogonadism

D. Fracture is more common on stratum compactum instead of trabecular.

E. Osteocyt : osteoblasts that are trapped in cortical bone during the remodeling
process.

35. Which the following are associated with an increased risk of osteoporosis?

A. Malabsorption syndrome

B. Steroid therapy

C. Hyperparathyroidism

D. Late menopause

E. Premature ovarian failure (POF)

36. Case-1
A 32-year-old nulliparous woman has had oligomenorrhea since menarche. During the
past 5 years she has experienced slow but progressive increase in hair on her face,
back and forearms in the inter-mammary space, and on the back of her hands. Her
voice has slowly deepened and temporal balding and clitoromegaly have developed.
Which of the following is the most likely diagnosis?

A. Polycystic ovary syndrome

B. Cushing’s syndrome

C. Stromal hyperthecosis

D. Ovarian tumor

E. Asherman’s syndrome

37. Case-2.

A couple have a 2-year history of infertility. The male partner is healthy with no past
serious illnesses, and has one child from his previous marriage. The female partner
has a regular 29-day menstrual cycle and has a normal body mass index. Her only
previous pregnancy was an ectopione. Which of the following investigations is most
likely to reveal the cause of the infertility?

A. Semen analysis

B. Pelvic ultrasound examination

C. Hormonal profile

D. Hysteroscopy

E. Laparoscopy

Case-3 (questions 38 – 40)

A 47-year-old patient presents wondering if her problems with mood, swings,


insomnia, and vaginal dryness represent menopause. She had a hysterectomy 10
years ago for abnormal uterine bleeding, but the uterus were not removed. Since she
cannot afford hormonal testing, a maturation index is done on her pap smear.

38. Ideally, cytologic cells for evaluation of hormonal status should be obtained from the

A. Ectocervix

B. Endocervix

C. Labia minora

D. Lateral vaginal wall

E. Posterior vaginal fornix.


39. Ninety percent of the cells found on her pap smear have thick, rounded cytoplasm
and plump, round, vesicular nuclei with an intact chromatin patter. The maturation
index (MI) would most likely be

A. 90 / 0 / 10.

B. 90 / 10 / 0

C. 10 / 0 / 90

D. 10 / 90 / 0.

E. 0 / 90 / 10

40. Based on this result, one would anticipate that if hormonal levels were obtained they
would show

A. Elevated estrogen level

B. Elevated progesterone levels.

C. Low estrogen levels

D. Normal estrogen levels

E. Normal progesterone levels

Case-4

A patient presented to the emergency department with an infected incomplete


abortion. During the dilatation and curettage (D&C) excessive bleeding developed that
required vigorous curetting to control. She returns to the physician 6 months later
complicating that she has not had a menstrual cycle. She has all the symptoms of
getting ready to start a period but never sees any bleeding.

41. This history implies that what layer of endometrium is damaged ?

A. Basal zone

B. Compact zone

C. Functional zone

D. Spongy zone

E. None of above

42. Infertility can be caused by infection of  


A. Staphylococcus

B. Streptococcus

C. Chlamydia

D. Candidaalbicans

E. Trichomonas

43. Müllerian agenesis

A. is the second most common cause of primary amenorrhea

B. is associated with an abnormal karyotype

C. no associated with spinal skeletal anomalies

D. requires laparoscopic diagnosis

E. no ovulation

44. Which of the following substances stimulates GnRH secretion?

A. Beta-endorphin

B. Dopamine

C. Dynorphin

D. Norepinephrin

E. Serotonin

45. Leiomyomata

A. Are suppress to grow when exposed to progestins

B. Are infrequently associated with endometrial hyperplasia

C. Can be reduced in size when treated with RU486

D. Are a frequent cause of infertility

E. Treatment option always hysterectomy

46. The three principal androgens in decreasing order of potency are

A. Androstenedione, testosterone,dihydrostestosterone

B. Dihydrostestosterone, androstenedione, testosterone

C. Dihydrostestosterone, testosterone, androstenedione

D. Testosterone, androstenedione, dihydrostestosterone

E. Testosterone,dihydrostestosterone, androstenedione
47. Which of the following is not used to correct anovulation?

A. Bromocriptine

B. Weight loss

C. HCG injection

D. Low-dose oestrogen

E. Clomiphen citrate

48. Which of the following may not cause male infertility?

A. Gonorrhoea infection

B. Cystic fibrosis

C. Prolactinoma

D. Hydrocele

E. Varicocele

49. Which of the following are associated with early pregnancy loss?

A. Sexual intercourse

B. Multiple partner

C. Wart virus infection

D. Pulmonal tuberculosis

E. Intra-uterine pregnancy with an intra-uterine device in situ

50. Regarding osteoporosis, which one is not true:

A. Hip fracture causes more deaths than cancer of the cervix, uterus and ovaries
combined

B. Bone densitometry can be used to screen individual at risk

C. May be associated with hypogonadism

D. Fracture is more common on stratum compactum instead of trabecular.

E. Osteocyt : osteoblasts that are trapped in cortical bone during the remodeling
process.
1. You see five postmenopausal patients in the clinic. Each patient hasone of the
conditions listed, and each patient wishes to begin hormonereplacement therapy
today. Which patient would you start on therapy atthe time of this visit?
a. Mild essential hypertension
b. Liver disease with abnormal liver function tests
c. Malignant melanoma
d. Undiagnosed genital tract bleeding
e. Treated stage III endometrial cancer

2. A 9-year-old girl presents for evaluation of regular vaginal bleeding.History


reveals thelarche at age 7 and adrenarche at age 8. The most commoncause of this
condition in girls is
a. Idiopathic
b. Gonadal tumors
c. McCune-Albright syndrome
d. Hypothyroidism
e. Tumors of the central nervous system

3.Which of the following is a true statement regarding the psychologicalsymptoms of the


climacteric?
a. They are considerably less important than hormone levels
b. They commonly include insomnia, irritability, frustration, and malaise
c. They are related to a drop in gonadotropin levels
d. They are not affected by environmental factors
e. They are primarily a reaction to the cessation of menstrual flow

4. Osteoporosis is least likely in which of the following women?


a. Asian
b. White
c. Smokers
d. Sedentary
e. Obese

5. Which of the following is consistent with a diagnosis of delayedpuberty?


a. Breast budding in a 10-year-old girl
b. Menarche delayed beyond 16 years of age
c. Menarche 1 year after breast budding
d. FSH values less than 20 mIU/mL
e. Obese

6.An 18-year-old consults you for evaluation of disabling pain with hermenstrual periods. The
pain has been present since menarche and isaccompanied by nausea and headache. History is
otherwise unremarkable,and pelvic examination is normal. You diagnose primary
dysmenorrhea
and recommend initial treatment with which of the following?
a. Ergot derivatives
b. Antiprostaglandins
c. Gonadotropin-releasing hormone (GnRH) analogues
d. Danazol
e. Codeine

7. Normal stature with minimal or absent pubertal development maybe seen in


a. Testicular feminization
b. Kallman syndrome
c. Pure gonadal dysgenesis
d. Turner syndrome
e. Intermittent athletic training

8.Medications used in the treatment of idiopathic central precociouspuberty include


a. Exogenous gonadotropins
b. Ethinyl estradiol
c. GnRH agonists
d. Clomiphene citrate
e. Conjugated estrogens (e.g., Premarin)

9.Delayed puberty and sexual infantilism associated with hypergonadotropichypogonadism


can be seen in patients with which of the following?
a. Adrenogenital syndrome (testicular feminization)
b. McCune-Albright syndrome
c. Kallman syndrome
d. Gonadal dysgenesis
e. Mullerian agenesis

10.While evaluating a 30-year-old woman for infertility, you diagnosea bicornuate uterus.
You explain that additional testing is necessarybecause of the woman’s increased risk of
congenital anomalies in whichorgan system?
a. Skeletal
b. Hematopoietic
c. Urinary
d. Central nervous
e. Tracheoesophageal

11.A 39-year-old woman, gravida 3, para 3, complains of severe, progressivesecondary


dysmenorrhea and menorrhagia. Pelvic examinationdemonstrates a tender, diffusely enlarged
uterus with no adnexal tenderness.Results of endometrial biopsy are normal. This patient
most likely has
a. Endometriosis
b. Endometritis
c. Adenomyosis
d. Uterine sarcoma
e. Leiomyoma

12.The most important indication for surgical repair of a double uterus,such as a septate or
bicornuate uterus, is
a. Habitual abortion
b. Dysmenorrhea
c. Menometrorrhagia
d. Dyspareunia
e. Premature delivery

13.In an amenorrheic patient who has had pituitary ablation for a craniopharyngioma,which
of the following regimens is most likely to result inan ovulatory cycle?
a. Clomiphene citrate
b. Pulsatile infusion of gonadotropin-releasing hormone (GnRH)
c. Continuous infusion of GnRH
d. Human menopausal or recombinant gonadotropin
e. Human menopausal or recombinant gonadotropin followed by human
chorionicgonadotropin (hCG)
14. In the evaluation of a 26-year-old patient with 4 months of secondaryamenorrhea, you
order serum prolactin and β-hCG assays. Thepregnancy test is positive, and the prolactin
comes back at 100 ng/mL (normal<25 ng/mL in this assay). This patient requires
a. Routine obstetric care
b. Computed tomography (CT) scan of her sellaturcica to rule out pituitary adenoma
c. Repeat measurements of serum prolactin to ensure that values do not increaseover 300
ng/mL
d. Bromocriptine to suppress prolactin
e. Evaluation for possible hypothyroidism

15. Which of the following medications is used as first-line therapy inthe treatment of
endometriosis?
a. Unopposed estrogens
b. Dexamethasone
c. Danazol
d. Gonadotropins
e. Parlodel

16. A 28-year-old nulligravid patient complains of bleeding between herperiods and


ncreasingly heavy menses. Over the past 9 months she hashad two dilation and curettages
(D&Cs), which have failed to resolve hersymptoms, and oral contraceptives and
antiprostaglandins have notdecreased the abnormal bleeding. Of the following options, which
is mostappropriate at this time?
a. Perform a hysterectomy
b. Perform hysteroscopy
c. Perform endometrial ablation
d. Treat with a GnRH agonist
e. Start the patient on a high-dose progestational agent

17.Danazol used in the treatment of endometriosis causes which of thefollowing changes


within the endometrium and endometriosis tissue?
a. Aplasia
b. Atrophy
c. Hyperplasia
d. Neoplasia
e. Inflammation

18. Which of the following conditions can be diagnosed with a hysterosalpingogram?


a. Endometriosis
b. Hydrosalpinx
c. Subserous fibroids
d. Minimal pelvic adhesions
e. Ovarian cyst

19.The presentation of Asherman syndrome typically involves


a. Hypomenorrhea
b. Oligomenorrhea
c. Menorrhagia
d. Metrorrhagia
e. Dysmenorrhea

20.During the evaluation of secondary amenorrhea in a 24-year-oldwoman,


hyperprolactinemia is diagnosed. Which of the following conditionscould cause increased
circulating prolactin concentration and amenorrheain this patient?
a. Stress
b. Primary hyperthyroidism
c. Anorexia nervosa
d. Congenital adrenal hyperplasia
e. Polycystic ovarian disease

21.Premenopausal peripheral conversion of estrogen precursors in theobese patient results in


the formation of
a. Estriol
b. Estradiol
c. Estrone
d. Androstenedione
e. Dehydroepiandrosterone

22.Varicoceles appear to cause male infertility by


a. Interfering with sperm production
b. Blocking epididymal sperm motility activation
c. Increasing the likelihood of sperm antibody formation
d. Interfering with sperm movement through cervical mucus
e. None of the above

23.The presence of a uterus and fallopian tubes in an otherwise phenotypicallynormal male is


due to
a. Lack of Mullerian-inhibiting factor
b. Lack of testosterone
c. Increased levels of estrogens
d. 46,XX karyotype
e. Presence of ovarian tissue early in embryonic development

24.Luteal phase defects are ovulatory disorders that can be a cause ofinfertility. Which of the
following studies performed in the second half ofthe menstrual cycle is helpful in making this
diagnosis?
a. Serum estradiol levels
b. Urinary pregnanetriol levels
c. Endometrial biopsy
d. Serum follicle-stimulating hormone (FSH) levels
e. Serum luteinizing hormone (LH) levels

25.A 45-year-old woman who had two normal pregnancies 15 and 18years ago presents with
the complaint of amenorrhea for 7 months. Sheexpresses the desire to become pregnant again.
After exclusion of pregnancy,which of the following tests is next indicated in the evaluation
of
this patient’s amenorrhea?
a. Hysterosalpingogram
b. Endometrial biopsy
c. Thyroid function tests
d. Testosterone and DHAS levels
e. LH and FSH levels
26.A 22-year-old woman consults you for treatment of hirsutism. She isobese and has facial
acne and hirsutism on her face and periareolar regionsand a male escutcheon. Serum LH level
is 35 mIU/mL and FSH is 9mIU/mL.Androstenedione and testosterone levels are mildly
elevated, butserum DHAS is normal. The patient does not wish to conceive at this time.
Which of the following single agents is the most appropriate treatment ofher condition?
a. Oral contraceptives
b. Corticosteroids
c. GnRH
d. Parlodel
e. Wedge resection

27.An 18-year-old college student who has recently become sexuallyactive is seen for severe
primary dysmenorrhea. She does not want to getpregnant, and has failed to obtain resolution
with heating pads and mildanalgesics. Which of the following medications is most
appropriate for thispatient?
a. Prostaglandin inhibitors
b. Narcotic analgesics
c. Oxytocin
d. Oral contraceptives
e. Luteal progesterone

28. Retrograde menstruation is the most accepted mechanism to explainthe etiology of


endometriosis. Another theory suggests that some stimuluscauses metaplasia of the celomic
epithelium, leading to endometriosis.Endometriosis in which of the following patients is
evidence of the celomicmetaplasia theory of causation?
a. A patient with endometriosis in an episiotomy scar
b. A patient with endometriosis of the subarachnoid space
c. A patient with endometriosis in the lung
d. A patient with Mullerian agenesis
e. A patient with endometriosis in a laparoscopy scar

29. A 19-year-old patient presents to your office with primary amenorrhea.She has normal
breast and pubic hair development, but the uterusand vagina are absent. Diagnostic
possibilities include
a. XYY syndrome
b. Gonadal dysgenesis
c. Mullerian agenesis
d. Klinefelter syndrome
e. Turner syndrome

30.Which of the following medications is most useful for the treatment of premenstrual
syndrome?
a. Progesterone
b. Anxiolytics
c. Vitamins
d. Antiprostaglandins
e. Selective serotonin reuptake inhibitors (SSRIs)

31.A 23-year-old woman presents for evaluation of a 7-month history ofamenorrhea.


Examination discloses bilateral galactorrhea and normalbreast and pelvic examinations.
Pregnancy test is negative. Which of thefollowing classes of medication is a possible cause of
her condition?
a. Antiestrogens
b. Gonadotropins
c. Phenothiazines
d. Prostaglandins
e. GnRH analogues

32.Which of the following pubertal events in girls is not estrogendependent?


a. Menses
b. Vaginal cornification
c. Hair growth
d. Reaching adult height
e. Production of cervical mucus

33. A 9-year-old girl has breast and pubic hair development. Evaluationdemonstrates a
pubertal response to a gonadotropin-releasing hormone(GnRH) stimulation test and a
prominent increase in luteinizing hormone(LH) pulses during sleep. These findings are
characteristic of patients with
a. Theca cell tumors
b. Iatrogenic sexual precocity
c. Premature thelarche
d. Granulosa cell tumors
e. Constitutional precocious puberty
34. Which of the following findings characterizes a normal semen sample?
a. Agglutination
b. Sperm concentration of 35 million per mL
c. 5% normal sperm morphology
d. 10% progressive sperm motility
e. A volume of 1 mL
QUESTION 35-39
For each evaluation, select the most appropriate day of a normal
28-day menstrual cycle for a woman with 5-day menstrual periods.
a. Day 3
b. Day 8
c. Day 14
d. Day 21
e. Day 26
35. Endometrial biopsy for evaluation of infertility (SELECT 1 DAY)E
36. Postcoital test (SELECT 1 DAY) C
37. Hysterosalpingogram (SELECT 1 DAY)B
38. Determination of serum progesterone level to document ovulationD
(SELECT 1 DAY)
39. Gonadotropin evaluation (SELECT 1 DAY)- A

A 22-year-old G0P0 comes to your office with a chief complaint ofbeing too hairy. She
reports that her menses started at age 13 and havealways been very irregular. She also
complains of acne and is currently seeinga dermatologist for the skin condition. She denies
any medical problems,and her only surgery was an appendectomy at age 8. Height is 5 ft,5
in.; weight is 150 lb; BP is 100/60. On physical exam, there is sparse hairaround the nipples,
chin, and upper lip. No galactorrhea, thyromegaly, ortemporal balding is noted. Pelvic exam
is normal and there is no evidenceof clitoromegaly.

40. All of the following should be included in the differential diagnosisbased on the patient’s
history and physical exam except
a. Idiopathic or constitutional hirsutism
b. Polycystic ovarian syndrome
c. Late-onset congenital adrenal hyperplasia
d. Sertoli-Leydig cell tumor
41.All of the following blood tests would be appropriate to order in theworkup of this patient
except
a. Total testosterone
b. 17 α-hydroxyprogesterone
c. DHEAS
d. Estrone
e. TSH
f. Prolactin

42. After all the appropriate lab studies are drawn, you conclude that thepatient has polycystic
ovarian syndrome. All of the following are appropriatetreatments for this disorder except
a. Dexamethasone
b. Oral contraceptives
c. Spironolactone
d. Metformin
e. Weight reduction
QUESTION 43-45
A patient in your practice calls you in a panic because her 14-year-olddaughter has
beenbleeding heavily for the past 2 weeks and now feels a bit dizzy and lightheaded. The
dauhter experienced menarche about 1 yearage, and since that time her periods have been
extremely irregular andunpredictable. You instruct the mother to bring her daughter to the
emergencyroom. When you see the daughter in the emergency room, you notethat she appears
very pale and fatigued. Her blood pressure and pulse are110/60 and 70, respectively. When
you stand her up, her blood pressureremains stable, but her pulse increases to 100. While in
the emergencyroom, you obtain a more detailed history. She denies any medical problemsor
prior surgeries and is not taking any medications. She reports that shehas never been sexually
active. On physical exam, her abdomen is benign.She will not let you perform a speculum
exam, but the bimanual exam isnormal. She is five ft tall and weighs 95 lb.
43.All of the following are appropriate lab tests to order in the emergencyroom except
a. BHCG
b. Bleeding time
c. CBC
d. Type and screen
e. Estradiol level

44. Which of the following is the most likely diagnosis?


a. Uterine fibroids
b. Cervical polyp
c. Incomplete abortion
d. Anovulation
e. Cervical cancer

45. While you wait for the lab work to come back, you order intravenoushydration. After 2 h,
the patient is no longer orthostatic. Her BHCG comesback negative, and her Hct is 22. What
is the best next step in the managementof this patient?
a. Perform a dilation and curettage
b. Administer a blood transfusion to treat her severe anemia
c. Send her home with a prescription for iron therapy
d. Administer high-dose oral estrogen therapy
e. Administer antiprostaglandins

46. The following statements regarding prolactin is true : (D)


A. It’s secreted by nucleus arcuatus
B. It stimulates oxytocn secretion
C. It stimulates GnRH secretion
D. The secretion is stimulated by TRH
E. It stimulates Norephineprin secretion

47. The following statements is true regarding the short loop feed back of GnRH.(E)
A. Sex steroid relationship to pituitary.
B. Conection of GnRH with it’s own secretion.
C. Conection of hypothalamus to pituitary
D. With the down regulation mechanism
E. Through feed back of hypophyseal portal system.

48. The following is true regarding GnRH: (B)


A. It’s a steroid hormone
B. It has membrane receptor
C. It enters posterior lobe of pituitary
D. It directly enters from tractus tubero infundibular.
E. The secretion is stimulated by dopamine

49. The following is true regarding GnRH (A)


A. The secretion is stimulated by norephineprin
B. Hypothyroid will stimulate it’s secretion.
C. Longterm and high dose of GnRH will increase multiple ovulation.
D. Pineal gland hormone stimulate it’s secretion
E. Oxytocin will increase it’s degradation.

50. The following is true regarding the secretion of GnRH (C)


A. The amplitudo is higher in early cycle than in the luteal phase.
B. In early cycle the amplitudo is higher than in the midcycle.
C. In midcycle, the frequency of secretion is increase
D. In midcycle the amplitudo is higher than in the luteal phase.
E. In luteal phase the frequency is higher than in the early cycle.
51. The following is true regarding early relationship of hypothalamus-pituitary-ovarian axis
C)
A. The first initiation is prior to puberty
B. Initiated with FSH increase.
C. At puberty, central hypersensitivity to estrogen is decline.
D. Soon will lead to regular ovulation.
E. Resulting in the first folliculogenesis.

52. The following is true regarding oocytes reserve (B)


A. The highest oocytes reserve is at the time of parturition.
B. The development of primordial follicle disappear for sometimeafter parturition.
C. At menarche there are 6 to 7 million oocytes.
D. Pregnancy will influence the decline of oocytes reserve.
E. Stimulation drugs inhance the decline of oocytes reserve.

53. The following is true regarding oogenesis (meiosis) ( D).


A. Oogonia has haploid chromosom
B. The first polar body (polar body I) disconnected at the early stage of meiosis I
C. Polar body II disconnected at the time of ovulation
D. Meiosis disappear during follicular phase
E. The final result is 4 oocytes

54. The following is true regarding spermiogenesis (E)


A. The development of diploid to haploid
B. Process of spermatogonia to primary spermatocyte.
C. Process of primary oocyte to secondary spermatocyte.
D. Process of secondary spermatocyte to spermatide.
E. Maturation process of spematide to spermatozoa.

55. The following is true regarding the situation of early cycle (E)
A. The LH receptor is in granulosa cells
B. FSH stimulates the expression of of LH receptor.
C. Estradiol and FSH stimulate LH receptor.
D. Progesteron stimulates the secretion center of gonadotropin
E. Estradiol inhibits the secretion center of gonadotropin

56. The following statements regarding prolactin is true : (D)


A. It’s secreted by nucleus arcuatus
B. It stimulates oxytocn secretion
C. It stimulates GnRH secretion
D. The secretion is stimulated by TRH
E. It stimulates Norephineprin secretion

57. The following statements is true regarding the short loop feed back of GnRH.(E)
A. Sex steroid relationship to pituitary.
B. Conection of GnRH with it’s own secretion.
C. Conection of hypothalamus to pituitary
D. With the down regulation mechanism
E. Through feed back of hypophyseal portal system.
58. The following is true regarding GnRH: (B)
A. It’s a steroid hormone
B. It has membrane receptor
C. It enters posterior lobe of pituitary
D. It directly enters from tractus tubero infundibular.
E. The secretion is stimulated by dopamine
59. The following is true regarding GnRH (A)
A. The secretion is stimulated by norephineprin
B. Hypothyroid will stimulate it’s secretion.
C. Longterm and high dose of GnRH will increase multiple ovulation.
D. Pineal gland hormone stimulate it’s secretion
E. Oxytocin will increase it’s degradation.

60. The following is true regarding the secretion of GnRH (C)


A. The amplitudo is higher in early cycle than in the luteal phase.
B. In early cycle the amplitudo is higher than in the midcycle.
C. In midcycle, the frequency of secretion is increase
D. In midcycle the amplitudo is higher than in the luteal phase.
E. In luteal phase the frequency is higher than in the early cycle.

61. The following is true regarding early relationship of hypothalamus-pituitary-ovarian


axis C)
A. The first initiation is prior to puberty
B. Initiated with FSH increase.
C. At puberty, central hypersensitivity to estrogen is decline.
D. Soon will lead to regular ovulation.
E. Resulting in the first folliculogenesis.

62. The following is true regarding oocytes reserve (B)


A. The highest oocytes reserve is at the time of parturition.
B. The development of primordial follicle disappear for sometimeafter parturition.
C. At menarche there are 6 to 7 million oocytes.
D. Pregnancy will influence the decline of oocytes reserve.
E. Stimulation drugs inhance the decline of oocytes reserve.

63. The following is true regarding oogenesis (meiosis) ( D).


A. Oogonia has haploid chromosom
B. The first polar body (polar body I) disconnected at the early stage of meiosis I
C. Polar body II disconnected at the time of ovulation
D. Meiosis disappear during follicular phase
E. The final result is 4 oocytes

64. The following is true regarding spermiogenesis (E)


A. The development of diploid to haploid
B. Process of spermatogonia to primary spermatocyte.
C. Process of primary oocyte to secondary spermatocyte.
D. Process of secondary spermatocyte to spermatide.
E. Maturation process of spematide to spermatozoa.

65. The following is true regarding the situation of early cycle (E)
A. The LH receptor is in granulosa cells
B. FSH stimulates the expression of of LH receptor.
C. Estradiol and FSH stimulate LH receptor.
D. Progesteron stimulates the secretion center of gonadotropin
E. Estradiol inhibits the secretion center of gonadotropin
Soal No. 1-3 Mrs N (25 y.o) with infertility, irregular cycles and hirsutism, 160 cm height,
BW 50 kg. 6 months ago Laparoskopic ovarian drilling was done, but there is no
improvement of her cycles. Both tubes was patent and no another abnormality. Sperm
analysis : concentration 4 million/mL, good motility 2%, normal morphology 1%, there is no
improvement after treated by andrology expert.
1. What the next treatment do you suggest. 
a. Intra uterine insemination.
b. IVF-ET
c. IVF-ICSI-ET.
d. IVF-PESA-ICSI-ET.
e. IVF-TESE-ICSI-ET

2. Which ovarian stimulation do you use ?


a. Chronic low dose rFSH with GnRHa short protocol.
b. Chronic low dose rFSH with GnRHa long protocol.
c. Step down rFSH with GnRHa short protocol.
d. Step up rFSH with short GnRHa protocol.
e. Chronic low dose rFSH with GnRH antagonist

3. To reduce risk, which regimen do you use for oocyte maturation before oocytes
retrievel ?  
a. rLH.
b. hCG.
c. GnRH antagonist.
d. GnRH agonist
e. Anti – PG

Soal No. 4-6 Miss T (25 yo) in the last 1 year every month get severe headache, irritability
emotion, abdominal discomfort, and can not work. These complain started at mid cycles and
stop when menstrual cycle started. Physical and gynecological examination in normal limit,
she has regular cycle.
4. What is the most possibility clinical diagnosis :  
a. Endometriosis.
b. Premenstrual syndrome.
c. Dysmenorrhea.
d. P I D.
e. Torsion ovarian cyst

5. What is the next examination for support the diagnosis ?  


a. TVS.
b. Laparoscopy.
c. Serum FSH.
d. Serum progesterone.
e. Anamnesis.

6. What is the first line treatment in this case.?  


a. Estradiol and drospirenone orally.
b. Gonadotrophin Releasing Hormone agonist depot.
c. Levonorgestrel orally.
d. Depomedroxy progesterone acetate.
e. Desogestrel implant

Soal No. 7-9 Miss P ( 29yo), consulted by GP, with irregular cycle, LMP 2 years ago. She got
menstruation if treated with combined oral contraception, menarche at 14 yo. 155 cm heigh,
55 kg BW. Blood pressure 110/70. Physical and gynecological examination within normal
limit.
7. Initial examination in this case is :  
a. Serum estradiol.
b. Serum FSH.
c. Serum Progesterone.
d. Serum prolactin.
e. Serum TSHs

8. If the result of initial examination is high, the next examination is,  


a. Fotosellatursica.
b. ACTH provation test.
c. Karyotiping.
d. Laparoscopy
e. Histeroscopy

9. This abnormality can caused by :  


a. Autoimmun disease
b. Pituitary adenoma.
c. Insulin resistance.
d. Curettage.
e. Emotional stress

Soal No. 10-12 Miss N 17 years old consulted by GP with severe dysmenorrhea suspected
endometriosis. She has regular cycles ( 28 days), no history of pelvic surgery, PID or sexual
activity. 155 cm heigh, 50 kg BW. Physical examination within normal limit. Gynecological
examination, RF immobile uterus with normal size, no adnexal mass. Abdominal USG no
abnormality.
10. The next management is,  
a. MRI
b. CT scan
c. Laparoscopy
d. Laparatomy.
e. COCs treatment

11. These statements are correct about endometriosis related to pain, EXCEPT.  
a. Caused by inflammatory process in the peritoneal cavity.
b. Caused by effect of local endometrial implant bleeding.
c. Caused by infiltration of nerve in the pelvic floor.
d. 5 years recurrence rate of pain, after GnRHa treatment approximately 10%.
e. Recurrence rate of pain after treatment with danazol or COCs is similar

12. This side effect have to consider in the medical treatment of adolescent endometriosis,  
a. The effect on bone metabolism
b. The risk of malignancy.
c. The risk of heavy menstrual bleeding.
d. The risk of cardiovascular disease.
e. The effect on glucose metabolism

Soal No. 13- 15 Mrs M, 30 yo, 5 years infertility, regular cycle, no abdominal pain, no history
of pelvic surgery. The result of HSG 2 year ago is normal, sperm analysis, normozoospermia.
4 times stimulated IUI was done, in the last 1 year period.
13. The appropriate next infertility management is,  
a. Saline Infuse Sonografy.
b. Hysteroscopy.
c. Laparoscopy.
d. Post Coital Test.
e. ASA examination

14. The reason of previous examination is,  


a. 60% of normal HSG is incorrect.
b. Post Coital Test has strong correlation with pregnancy rate.
c. ASA is the most frequent factor in sperm infertility.
d. HSG can not excluded peritoneal factor.
e. Uterine factor is dominant factor in infertility problem

15. If in the result of previous examination is normal, the next step is  


a. ICSI.
b. PESA.
c. TESE.
d. Adoption.
e. IVF-ET

Soal No. 16-18 Miss P 35 yo, consulted by GP with amenorrhea, LMP 8 months ago. In the
last 2 years, her cycle become irregular, has growing new hair at chin, chest, surround
umbilicus and acne. No history of taking any drug/medication. Physical examination, blood
pressure : 170/100, pulse rate 100/menit, 160 cm height, 90 kg body weight, with moon face
and hirsutism . Other physical and gynecological examination within normal limit.
16. Base on clinical data the most probability diagnose is,  
a. PCOs.
b. Non classic CAH.
c. Cushing’s syndrome
d. Hyperthecosis.
e. Insulin resistance.

17. The important basic clinical data for previous clinical diagnose is  
a. Hirsutism and moon face.
b. Obesity and Hypertension
c. Obesity and amenorrhea.
d. Amenorrhea and hypertension.
e. Irregular cycle and moon face

18. To confirm the diagnosis the next examination is,  


a. Cortisol level.
b. Fasting serum insulin
c. Serum DHEA
d. Free testosterone.
e. Serum LH

Soal No. 19-21 Miss M (15 yo) consulted by GP, with irregular cycles, she get bleeding for a
month. No history of medication or chronic disease, she got menarche at 14 yo. Blood
pressure 110/70, pulse rate 80/m, Hb. 7 g%. 150 cm height, 50 kg weight. Physical and
gynecological examination within normal limit, only slight bleeding from the vagina.

19. The causal of prolonged and heavy bleeding in this case is,  
a. Blood coagulation defect.
b. Simultaneous endometrial shedding.
c. Atonic uterine contraction.
d. Endometrial atrophy
e. Fragile endometrium with a lot of microvascular

20. Pathophysiology of Abnormal Uterine bleeding in this case, caused by,  


a. Lower FSH secretion
b. Absent of LH surge.
c. Absent of follicles cyclic recruitment.
d. Lower estradiol production.
e. Elevated androgen

21. The most appropriate treatment beside improve general condition is,  
a. GnRHa depot.
b. DMPA
c. High dose COCs
d. Progestin oral.
e. NSAIDs

Soal No. 22-24 Mrs Y 46 yo, with irregular cycle, 3 living children, no history of
contraception or medication. LMP 6 months ago, 160 cm height, 60 kg weight, blood pressure
130/80, pulse rate 80/m, pregnancy test with negative result. Physical and gynecological
examination within normal limit. Serum FSH 15 mIU/mL, Estradiol 70 pg/mL.
22. The diagnose is,  
a. Menopause.
b. Oligo ovulation.
c. Premature Ovarian Insufficiency.
d. Perimenopause.
e. Post menopause

23. The earliest change in elderly women is,  


a. Declining serum AMH.
b. Elevated serum FSH.
c. Elevated serum LH
d. Declining serum estradiol.
e. Declining serum inhibin-B

24. The risk of osteoporosis increase if serum estradiol below,  


a. 10 pg/mL
b. 20 pg/mL
c. 30 pg/mL
d. 40 pg/mL
e. 50 pg/mL

25. Complication as a result of using gonadotropin for ovulation induction include  


a. No increased risk of spontaneous abortion
b. A 25 % risk of developing ovarian cancer
c. Less than 10 % multiple pregnancy
d. A 1 – 2 % risk of serious ovarian hyperstimulation
e. Increased risk of congenital anomaly

26. Proper monitoring for ovulation induction while using gonadotropin includes:  
a. Serum estradiol level
b. Serum progesterone level
c. Serum LH level
d. Serum Prolactin level
e. Serum FSH level

27. Ovarian hyperstimulationsyndrome  


a. Occurs in its moderate to severe form in more than 10 % of cases
b. Can be avoided by administration of hCG
c. It most likely to occur in women with polycyctic ovaries
d. It most likely to occur in women with endometriosis
e. Can be avoided by using a GnRH agonist in combined therapy

28. The addition of GnRH agonist to ovulation induction protocol in preparation for ART
have resulted in  
a. increased cancellation rates
b. Decreased oocyte yields
c. Increased pregnancy rates
d. Increased pregnancy lose
e. Decreased expense of cycle

29. To decrease the incidence of multiple births resulting from IVF  


a. The number of transferred embryos should be limited based upon the quantity of the
embryos
b. The number of transferred embryos should be limited based upon the age of the
women
c. Consideration of transferring a single blastocyst may be not an option
d. Selective fetal reduction may be not an option
e. All above are true

30. Ovarian endometriosis:  


a. Ovarian endometriomas occur more frequently on the left than the rightside
b. Most endometriomas occur as an intraovarian cyst.
c. Transvaginalsonography is a useful tool to make and to exclude an ovarian
endometrioma.
d. Small ovarian endometriomas are easily detected at laparoscopy.
e. Complete excision of a large endometriotic cyst guarantees that all endometriotic
tissue has been removed.

31. Regarding endometriosis:  


a. Decrease enzyme P-450 arom
b. Increase of LH level
c. Increase of androstenedion production
d. Increase local production of estrogen in the normal endometrium cell
e. Increase local production of estrogen in endometriosis cells
32. A 28 year old woman complain of infertility and is otherwise well. She had been on a
drug (haloperidol) to treat aschizophreni form illness she had for six years. She has a
healthy 3 years old daughter. What you suggest the cause of her secondary infertility?  
a. Hyperprolactinaemia
b. Endometriosis
c. Adenomyosis
d. Chronic salpingitis
e. Hypopituitarism

33. In a young, obese, chronically anovulatory woman with an elevated LH:FSH


(luteinizing hormone: follicle stimulating hormone) ratio and polycystic-appearing
ovaries, which of the following is the preferred initial method of ovulation induction?  
a. Metformin
b. Human menopausal gonadotropins (hMGs)
c. Pulsatile gonadotropin-releasing hormone (GnRH)
d. Clomiphene citrate
e. Bromicriptinemesylate

34. Which of the following is the best description of the mechanism of intrauterine
synechiae( Asherman’s syndrome)  
a. Pituitary engorgement
b. Myometrial scarring
c. Trophoblastic hyperplasia
d. Disruption of large and deep segments of the endometrium
e. Endometrial displasia

35. To preserve the pneumoperitoneum when placing secondary trocars, it is important to:  
a. Ensure the surgeon is in the proper position
b. Close the stopcock on the secondary port before insertion
c. Ensure the stopcock on the secondary port is open before insertion
d. Ensure the blade on the trocar is sharp
e. Close the insufflation stopcock on the primary port

36. During oocyte maturation, fully grown oocytes undergo structural and functional
modifications that allow them to continue fertilization and development. To proceed
monospermic fertilization:  
a. Two spermatozoa will fertilize an oocyte in order to develop twins pregnancy
b. Globozoospermia is sperm with long thin head and small acrosome area
c. Changes throughout oogenesis named redistribution of cortical granules originating
in the Golgi apparatus, allowing diploid outcome of fertilization
d. Only telophase completed oocyte of second meiosis could be fertilized
e. Nuclear maturity of the oocyte characterized by second polar body extrusion

37. In Hysterosalpingography, all are true EXCEPT:  


a. It is ideally performed postmenstrual
b. Failure to opacify the tubes may be due to uterine cornual spasm
c. A contrast medium with high viscosity is desirable
d. Filling of endocervical glands is a normal finding
e. Venous intravasation is a recognized complication

38. A 19-year-old woman presents with a complaint of 7 months of amenorrhea. She notes
that she hasn’t had a period since 2-3 months after starting college. She notes weight
loss during that time from 65 kgs to 59 kgs over the past few months but believes the
weight loss is secondary to change in diet during college. She also has insomia, heat
intolerance, and occasonally hot flashes. Which of the following tests is most likely to
indicate her diagnosis?  
a. Thyroid stimulating hormone
b. Luteinizing hormone
c. Prolactin
d. β-hCG
e. ACTH stimulation

39. The principal requisite for surfactant biosynthesis :  


a. IGFBP – 1
b. IGFBP – 2
c. IGFBP – 3
d. Fetal Cortisol
e. DHAS

40. Fetal gonadotropin:  


a. Are peak at 36weeks in uterus
b. Are higher in males than females probably due to testosterone and inhibin
c. Begin to produced in the pituitary by 32 weeks of intrauterine life
d. Peak after maximum number of oogonia and oocytes are produce in uterus
e. Are peak at 28 weeks in uterus

41. Leptin and reproduction  


a. Leptin administration does not accelerates the onset of puberty in rodents.
b. Leptin levels decrease at puberty.
c. Low leptin levels are present in athletes and in patients which anorexia.
d. Leptin levels are greater in males than females.
e. Leptin levels are increase in postmenopausal women

42. Which of the following is likely correct for PCOS patients :  


a. AMH level is higher than non PCOS
b. Decreased circulating insulin levels in obese patients
c. Decreased LH level leads to ovarian androgen production
d. Increased risk for developing liver disease
e. Weight reduction is the worst treatment for obese women

43. The following immunological functions are suppressed in women with endometriosis:
a. NK function.
b. Cytokine synthesis.
c. Antibody synthesis.
d. Phagocytosis.
e. Leukocyte chemotaxis 
44. A 16 year old girl has not experienced menarche. Examination shows absence of breast
development and small but otherwise normal female pelvic organs. Which of the
following diagnostic tests is most useful in determining the etiology of the amenorrhea?
a. FSH
b. Serum estradiol
c. Serum testosterone
d. MRI of the head
e. Ovarian biopsy

45. Normal fetal gonadotropins are necessary to control and coordinate the phenotypic
sexual differentiation of the fetus during intrauterine development. Which of the
following statements best describes circulating gonadotropin levels in the human fetus?
a. They are low during the first trimester, rise to maximal levels during the second
trimester, and return to low levels by term
b. They are low during the first trimester, rise to maximal levels during the second
trimester, and remain elevated to term.
c. They are low during the first trimester remain low during the second trimester and
rise to maximal levels by term.
d. They are high during the first trimester decrease to undetectable levels during the
second trimester, and remain low to term
e. They are low throughout intrauterine life.

46. Which of the following causes of delayed puberty accompanies elevated circulating
gonadotropin levels?  
a. Chronic illness
b. Gonadal dysgenesis
c. Hypothalamic tumors
d. Kallmann’s syndrome
e. Malnutrition

47. A 7-year-old girl is brought in for evaluation. On examination, she has well-developed
pubic hair and breasts. She is at 99% of height for her age. Her mother recently noted
some blood stains on her underwear. Which of the following conditions is most likely
the cause of these findings?  
a. Estrogen-producing ovarian cyst
b. Hepatoma
c. Hypothalamic tumor
d. Sex steroid-containing medication
e. Thecal/leydig cell tumor

48. A 47-year-old patient presents wondering if her problems with mood swings, insomnia,
and vaginal dryness represent menopause. She had a hysterectomy 10 years ago for
abnormal uterine bleeding, but the ovaries were not removed. Since she cannot afford
hormonal testing, a maturation index is done on her Pap smear. Ideally, cytologic cells
for evaluation of hormonal status should be obtaines from the:  
a. Ectocervix
b. Endocervix
c. Labia minora
d. Lateral vaginal wall
e. Posterior vaginal fornix
49. Even after menopause most women have circulating estrogen. In high enough levels,
this can promote the development of endometrial cancer. It mainly originates from the
aromatization of:  
a. Androstenedione to estrone by ovarian granulose cells
b. Androstenedione to estrone by ovarian theca cells
c. Androstenedione to estrone by adipose tissue
d. Testosterone to estradiol by adipose tissue
e. Estradiol to estrone by adipose tissue

50. A 50-year-old woman presents to her health care provider complaining of hot flushes.
Hot flushes are often the symptoms is an perimenopausal woman that causes her to
seek medical assistance. Hot flushes entail:  
a. Peripheral redistribution of blood flow leading to sweating and elevated heart rate
b. Peripheral vasodilatation reflecting an increase in core body temperature
c. Subjective symptoms always accompanying objective signs of vasomotor instability
d. Peripheral vasodilatation resulting from a direct LH action on sympathetic neurons
e. An average duration of about 30 minutes

51. This postmenopausal patient is interested in hormone replacement therapy (HRT) with
progesterone but is concerned about its dangers. Which of the following statements
should be included in you discussion regarding the risks of HRT with combined
therapy relative to no HRT?
a. Just as oral contraceptives may increase blood coagulability, HRT will also due to
higher doses
b. HRT may increase the risk of cholelithiasis
c. HRT may increase the risk of endometrial carcinoma
d. HRT is likely to greatly increase the risk of breast carcinoma
e. HRT may increase the risk of renal dysfunction

52. During normal pregnancy, which of the following physiologic effects occur?
a. Increased serum beta globulins (transport proteins) and decreased triglycerides
b. Increased serum corticosteroid-binding globulin and free cortisol
c. Increased levels of immunoglobulins A, G, and M
d. Increased thyroid-binding globulin and iodide levels
e. Decreased serum ionized calcium levels and parathyroid hormone (PTH)

53. A 28 year oldnulligravid patient complains of bleeding between her periods and
increasingly heavy menses. Over the past 9 months she had two dilation and curettages
(D&Cs), which have failed to resolve her symptoms, and oral contraceptives and
antiprostaglandins have not decreased the abnormal bleeding. Of the following options,
which is most appropriate at his time?  
a. Perform a hysterectomy
b. Perform hysteroscopy
c. Perform endometrial ablation
d. Treat with a GnRH agonist
e. Start the patient on a high-dose progestational agent

54. Mrs. F, 33 year-old with a microadenoma of the pituitary gland becomes pregnant .
When she reaches 14 week’s gestation, she complains of headaches, visual
disturbances. Wich of the following is the best therapy ?  
a. Transsphenoid pituitary resection
b. Thamoxifen therapy
c. Expectance management
d. Oral bromocriptine therapy
e. Regiment oxytocin and antidiuretic hormone

55. Diagnostic evaluation of abnormal bleeding, UNLESS :  


a. Indication of biopsy endometrium such as endometrial thickness >10 mm.
b. The need a sensitive pregnancy test.
c. a well-timed serum progesterone determination, can help to document ovulation or
anovulation.
d. If a very thin endometrial stripe (less than 5 mm), suggests an attenuated or denuded
endometrium.
e. Hysteroscopy is the definitive methode for intrauterine pathology.

56. Regarding endoscopy gynecology, which one of the following statements is


INCORRECT?  
a. A hemodynamically unstable patient with the need for control of bleeding probably
should be approached by laparotomy
b. A trick that provide safety in gaining access to the abdomen is by avoiding
Trendelenburg position
c. It’s safe to use two different energy sources in the abdomen at the same time
d. If the pressure used for peritoneal insufflation is limited, laparoscopy can be
performed under conscious sedation
e. Atraumatic forceps are used where trauma of the tissue are to be particularly
considered

57. Gonadotropin-releasing hormone (GnRH) stimulates the release of:  


a. Adrenocorticotropic hormone (ACTH)
b. Growth hormone (GH)
c. Luteinizing hormone (LH)
d. Opiate peptide
e. Thyroid-stimulating hormone (TSH)

58. Considering learning curve of a laparoscopic surgeon, these procedures should be


considered difficult cases, EXCEPT:  
a. Ovarian cyst size 4 cm with history of laparoscopic cholecystectomy
b. Uterine fibroid size more than 5 cm
c. Multiple uterine fibroid
d. Adenomiosis for hysterectomy
e. Deep infiltrating endometriosis
59. To preserve the pneumoperitoneum when placing secondary trocars, it is important to:  
a. Ensure the surgeon is in the proper position
b. Close the stopcock on the secondary port before insertion
c. Ensure the stopcock on the secondary port is open before insertion
d. Ensure the blade on the trocar is sharp
e. Close the insufflation stopcock on the primary port

60. The process of ovarian ageing imply for a testing prior to Assisted Reproductive
Technology  
a. Female age is a poor predictor for ovarian reserve.
b. Ovarian reserve can be considered normal when use of exogenous gonadotropins
result in the development of 1-2 follicles.
c. The endocrine marker anti-Mullerianhormon (AMH) which is produced by the
granulosa cells sorrounding the antral follicles, provides a novel direct marker of
quantity.
d. Basal FSH levels will become decreased with advancing age. Basal FSH provides
the most indirect marker.
e. The antral follicle count (AFC) assessed by laparoscopy provides direct visual
assessment of the cohort.

61. Which of the following statements regarding GnRH stimulated LH secretion is


accurate?  
a. It is enhanced by gonadotrope exposure to progesterone
b. It is increased by gonadotrope exposure to testosterone
c. It is enhanced by gonadotrope exposure to estrogen
d. It is enhanced by gonadotrope exposure to continuous GnRH
e. It is associated with steady LH release

62. Low density lipoprotein (LDL) cholesterol serves as the principal substrate for
steroidogenesis. Which of the following statements regarding circulating LDL is
correct?  
a. LDL is formed after addition of triglycieride to very low-density lipoprotein
(VLDL)
b. LDL levels are negatively correlated with cardiovascular disease
c. LDL is the major carrier of cholesterol in the plasma
d. LDLD enters the cells by passive diffusion
e. LDL facilitates the transport of polar lipids in the blood plasma

63. Which of the following events that can lead to antispermantibodies ?


a. pulmonary tuberculosis
b. nephrotic syndrome
c. lupus eritematous
d. hepatitis
e. parotitis 

64. Which of the following events that can lead to azoospermia with the result of Fructose
test (-) in semen?  
a. Congenital A VasDeferens (CAVD)
b. Orchitis
c. Klinefelter syndrome
d. Chronic Prostatitis
e. Varicocele

65. Normal stature with minimal or absent pubertal development maybe seen in  
a. Testicular feminization
b. Kallman syndrome
c. Pure gonadal dysgenesis
d. Turner syndrome
e. Intermittent athletic training

66. Medications used in the treatment of idiopathic central precociouspubertyinclude  


a. Exogenous gonadotropins
b. Ethinyl estradiol
c. GnRH agonists
d. Clomiphene citrate
e. Conjugated estrogens

67. While evaluating a 30-year-old woman for infertility, you diagnoseabicornuate uterus.
You explain that additional testing is necessary because of the woman’s increased risk
of congenital anomalies in which organ system?  
a. Skeletal
b. Hematopoietic
c. Urinary
d. Central nervous
e. Tracheoesophageal
68. A 39-year-old woman, gravida 3, para 3, complains of severe, progressivesecondary
dysmenorrhea and menorrhagia. Pelvic examinationdemonstrates a tender, diffusely
enlarged uterus with no adnexal tenderness.Results of endometrial biopsy are normal.
This patient most likely has  
a. Endometriosis
b. Endometritis
c. Adenomyosis
d. Uterine sarcoma
e. Leiomyoma

69. A 45-year-old woman who had two normal pregnancies 15 and 18years ago presents
with the complaint of amenorrhea for 7 months. She expresses the desire to become
pregnant again. After exclusion of pregnancy,which of the following tests is next
indicated in the evaluation of this patient’s amenorrhea?  
a. Hysterosalpingogram
b. Endometrial biopsy
c. Thyroid function tests
d. Testosterone and DHAS levels
e. FSH & Estradiol levels

70. A 23-year-old woman presents for evaluation of a 7-month history of amenorrhea.


Examination discloses bilateral galactorrhea and normal breast and pelvic
examinations. Pregnancy test is negative. Which of the following classes of medication
is a possible cause of her condition?  
a. Antiestrogens
b. Gonadotropins
c. Phenothiazines
d. Prostaglandins
e. GnRH analogues

71. A 9-year-old girl has breast and pubic hair development. Evaluation demonstrates a
pubertal response to a gonadotropin-releasing hormone(GnRH) stimulation test and a
prominent increase in luteinizing hormone (LH) pulses during sleep. These findings are
characteristic of patients with  
a. Theca cell tumors
b. Iatrogenic sexual precocity
c. Premature thelarche
d. Granulosa cell tumors
e. Constitutional precocious puberty

72. The following is true regarding early relationship of hypothalamus-pituitary-ovarian


axis  
a. The first initiation is prior to puberty
b. Initiated with FSH increase.
c. At puberty, central hypersensitivity to estrogen is decline.
d. Soon will lead to regular ovulation.
e. Resulting in the first folliculogenesis.

73. Woman with the gonadal dysgenesis :  


a. Have normal serum FSH level between age 8-16 years
b. Have normal serum FSH level between age 2-6 years
c. Have lowest serum FSH level consistent with normal menopaused values a
teenagers
d. Demonstrate augmented gonadotropin secretion during wake up
e. Have normal serum LH level between age 8-16 years

74. A couple with male infertility characterized by a semen analysis with a sperm count of
14 milion/mL, 25% motility, and 23% normal forms presents to your clinic. The
husband’s physical examination and hormone studies are normal. The appropriate
initial therapy is :  
a. Clomiphene citrate
b. Varicocelectomy
c. In vitro fertilization (IVF)
d. Intrauterine insemination with washed husband’s sperm
e. Fallopian sperm tube perfusim

75. A 9-year-old girl presents for evaluation of regular vaginal bleeding.History reveals
thelarche at age 7 and adrenarche at age 8. The most commoncause of this condition in
girls is  
a. Idiopathic
b. Gonadal tumors
c. McCune-Albright syndrome
d. Hypothyroidism
e. Tumors of the central nervous system

76. Which of the following pubertal events is not mediated by gonadal estrogen production
and therefore would occur even in the absence of estrogen production?  
a. Breast development
b. Menstruation
c. Pubic hair growth
d. Skeletal growth
e. Vaginal cornification

77. During an evaluation for infertility, a woman may have an endometrial biopsy to
evaluate the quality of her ovulation since the development of the corpus luteum is
most closely associated with the:  
a. Fertilization of an ovum
b. Follicular phase of the endometrium
c. Proliverative phase of the endometrium
d. Secretory phase of the endometrium
e. Shedding phase of the endometrium (menstruation)

78. Which statement best describes estrogen positive feedback on LH release?  


a. It is affected by the level of circulating estrogen
b. It is enhanced by testosterone
c. It is increased by opioid peptides
d. It is unaffected by progesterone
e. It is unaffected by the duration of estrogen stimulation

79. The simplest, yet most useful, initial test to begin her evaluation would be  
a. Serum estrogen level
b. Prolactin
c. Thyroid index
d. Serum follicle-stimulating hormone (FSH) and luteinizing hormone (LH)
e. A cardiogram

80. The histology of adenomyosisshows  


a. The metaplastic change of glandular epithelium to muscle fibers in the uterus
b. The same pattern and location as endometriosis
c. The presence of endometrial glands and stroma deep within uterine muscle
d. A premalignant change of the endometrium
e. A premalignant change of the uterine muscle

81. An increased risk of Osteoporosis is associated with:  


a. Bromocriptine use
b. Excess glucocorticosteroid usage
c. Androgen excess in the female.
d. Early menarche
e. Excessive protein intake

82. Recognized feature of postmenopausal women is:  


a. Low FSH & LH
b. Increased bone mineral density.
c. High FSH
d. High level of estrogen
e. Increase the incidence of uterine fibroid

83. Which of the following is a true statement regarding the psychological symptoms of the
climacteric?  

a. They are considerably less important than hormone levels.


b. They commonly include insomnia, irritability, frustration, and malaise
c. They are related to a drop in gonadotropin levels
d. They are not affected by environmental factors
e. They are primarily a reaction to the cessation of menstrual flow

84. Which of the following contraceptive methods should NOT be used by a patient with
coronary heart disease?  
a. Combined oral contraceptive pills
b. Male condom
c. Female condom
d. Diaphragm
e. Spermicidal agent

Soal No. 85-87 Mrs N 28 yo, with 5 years infertility, prolonged cycle, LMP 2 months ago.
She treated with MPA 10 mg twice daily for 10 days, but there was not bleeding. Sperm
analysis normozoospermia. Her body weight is 90 kg, with 155 cm height, blood pressure
160/100, and pulse 80/menit. her pubic hair growth upward until umbilicus. Gynecological
examination within normal limit. The result of urine pregnancy test was negative.
85. Based on clinical data the most possible diagnose of this case is, infertility with,  
a. Unovulation WHO group I and obesity.
b. Unovulation WHO group II and obesity
c. PCOs, obesity, and metabolic syndrome
d. Non classic CAH
e. Cushing’s syndrome
86. To confirm the previous clinical diagnose these examination have to do.  
a. Serum LH.
b. Serum FSH.
c. Serum Free Testosterone
d. Serum DHEAS.
e. TVS.

87. The first line regimen for ovulation induction is,  


a. Clomiphen citrate
b. Aromatase inhibitor
c. Recombinant FSH
d. Metformine
e. Laparoscopic Ovarian Drilling

Soal No. 88-90 Mrs P 40 yo with prolonged bleeding, consulted by GP. She has no child.
There is history of chronic abnormal uterine bleeding, no history of contraception or
medication. Physical examination :155 cm height, 80 kg weight, blood pressure 160/100,
pulse rate 100/menit, Hb 9 g%. Gynecological examination within normal limit. TVS uterus
in normal size, endometrial thickness 14 mm, PCO feature on both side of ovary.
88. The initial management in this case is.  
a. Progestin treatment.
b. High dose COCs
c. Estrogen high dose, followed by COCs
d. Curettage with histopathological examination
e. GnRHa depot
89. This statement is correct about hormonal treatment in this case, EXCEPT  
a. Progestin enhance conversion estradiol to estrone
b. Estrogen has coagulation effect
c. GnRHa depot can stop bleeding faster
d. The goal is estrogen-progestin withdrawal bleeding
e. Against oncogenic effect of estradiol

90. This case has high risk of,  


a. Malignancy of the ovary
b. Malignancy of myometrium
c. DM type I
d. Metabolic syndrome
e. Early menopause
Soal No.; 91-93 Mrs D 35 yo has 2 children use low dose COCs started 2 months later,
consulted by GP with intermenstrual spotting. No history of missing pill, also no history of
other medication. She taking COCs contained 20 mcg estradiol combined with 3 mg
drospirenone, 24 pills and 4 placebo. Blood pressure 130/80, pulse rate 80/m, 160 cm height,
60 kg weight. Physical examination and gynecological examination within normal limit
91. The most possibility causal of intermenstrual spotting in this case is,  
a. Very low dose of estradiol.
b. The side effect of drospirenone
c. The composition of packed 24 pills and 4 placebo
d. Accidental serum FSH increase
e. Accidental serum LH increase.

92. These statement is correct about this COCs, EXCEPT,  


a. Drospirenone against the LH surge
b. Drospirenone have anti androgenic effect
c. Estrogen component enhance the endometrial stability
d. Estrogen component cause increasing serum potassium
e. Lower dose estrogen against the blood coagulation effect

93. Benefit of this COCs is, EXCEPT  


a. Not cause dyslipidemia
b. Not increase retain of water
c. Improve elevated blood pressure
d. High effectiveness
e. Improve heavy menstrual bleeding

94. The following is true regarding the situation of early cycle  


a. The LH receptor is in granulosa cells
b. FSH stimulates the expression of of LH receptor
c. Estradiol and FSH stimulate LH receptor
d. Progesteron stimulates the secretion center of gonadotropin
e. Estradiol inhibits the secretion center of gonadotropin
95. The following is true regarding oocytes reserve  
a. The highest oocytes reserve is at the time of parturition
b. The development of primordial follicle disappear for sometimeafter parturition.
c. At menarche there are 6 to 7 million oocytes
d. Pregnancy will influence the decline of oocytes reserve
e. Stimulation drugs inhance the decline of oocytes reserve

96. Which of the following is the best explaination for breast development in a patient with
androgen insensitivity ?  
a. Gonadal production of estrogen
b. Adrenal production of estrogen
c. Breast tissue sensitivity to progesterone
d. Peripheral convertion of androgens
e. Autonomous production of breast-specific estrogen

97. Steroid hormones are classifies as:  


a. Amino acids
b. Phospholipids
c. Lipids
d. Glycoprotein
e. None of the above

98. Leptin and reproduction  


a. Leptin administration does not accelerates the onset of puberty in rodents.
b. Leptin levels decrease at puberty.
c. Low leptin levels are present in athletes and in patients which anorexia
d. Leptin levels are greater in males than females
e. Leptin levels are increase in postmenopausal women

99. While in the in vivo environment embryo is maintained their viability by cell
sorrounding in the fallopian tubes, in vitro condition of human embryos:  
a. Glucose is needed in large amount of cleavage stage human embryo
b. Genomic imprinting of the human embryo take place at pronuclear stage of
development
c. Glucose is not an important component in the culture system since human embryo
are using mainly amino acid for their energy source
d. Human embryo exhibits a considerable degree of plasticity enabling it to develop
under a wide variety of culture condition
e. Human culture system is performed in room temperature with sterile environment

100. Low estriol level in pregnancy correlates with;  


a. Fetal macrosomia
b. P450c17 deficiency
c. Adrenal hypofunction
d. Adrenal hyperfunction
e. 5-OH-steroid dehydrogenase deficiency
1. The differences between RNA and DNA are, except :

f. RNA uses pentose sugar


g. RNA is single stranted
h. RNA contain uracil
i. Sugar in RNA is ribose
j. RNA carries codon

What happen in increase Corticotrophin Releasing Hormone (CRH) in fetal development, All
statement in bellow are true; except :

A. Increase fetal cortisol is positive feedback relationship

B. Augment fetal ACTH secretion.

C. Adjunct progesterone secretion.

D. More DHEAS.

E. Increase estrogen

2. Which regard of the following is related to the first sign of pubertal development of
female :
f. Menarche
g. Menopause
h. Pubarche
i. Menstruation
j. Telarche

3. Mechanism of action for steroid hormones :

A. Rapidly transported across the cell membrane by osmose action.

B. All of which require direct interaction with DNA.

C. Via hormone-receptor complex to bind hormone-responsive element in DNA.

D. Not regulate posttranscriptional events.

E. Translation via DNA methylation.

4. This statement is not true about basic mechanisms of menstruation

f. Menstruation is preceeded by intense vasoconstriction of the spiral arterioles


g. Ischaemia induced by intense vasoconstriction of 4 to 24 hours will be
augmented by increased release of free oxygen radicals produced by tissue
(endometrial) desquamation
h. Platelet fibrin plug formation is grossly deficient in endometrium
i. Ruptured spiral arterioles do not form platelet fibrin thrombi in the early
phases of menstruation
j. The rapid formation of fibrin plugs and their degradation play a part of the
control of menstrual bleeding

5.The purpose of the progestational challenge test is to asses the level of endogenous :
F. Testosterone
G. Progesterone
H. Estrogen
I. Prolactin
J. TSH

6. Which of the following sequences best describes estrogen action?

a. cell membrane diffusion, steroidreceptor,-DNA complex formation, transcription,


translation

b.cell membrane receptor activation, steroidreceotor DNA complex formation,


translation,transcription, transcription

c. cell membrane diffusion' steroidreceptor-DNA complex formation, translation,


transcription

d.cell membrane diffusion, adenylatecyclase activation, cAMP production, protein


phosphorylation

e. cell membrane receptor activation, adenylatecyclase activation, c AMP production,


protein phosphorilation..

7.Amenorhea should be evaluated in any patient without a periode who has an absence of
growth or development of secondary sexual characteristic by age
F. 8
G. 10
H. 12
I. 14
J. 16

8.Sperm capacitation refers to a process by which spermatozoa become capable of:

a. stimulating meiosis of ovum

b. dispersing the zona radiata

c. penetrating the cervical mucus

d. producing acrosomal enzymes

e. fertilizing the ovum

9.Gonadotropin releasing hormone (GnRH) stimulates the release of:

f. Adrenocorticotropic hormone (ACTH)


g. Growth hormone (GH)
h. Luteinizing hormone (LH)
i. Opiate peptide
j. Thyroid-stimulating hormone (TSH)

10. This statement is not true about basic mechanisms of menstruation

a. Menstruation is preceeded by intense vasoconstriction of the spiral arterioles

b. Ischaemia induced by intense vasoconstriction of 4 to 24 hours will be augmented by


increased release of free oxygen radicals produced by tissue (endometrial)
desquamation

c. Platelet fibrin plug formation is grossly deficient in endometrium

d. Ruptured spiral arterioles do not form platelet fibrin thrombi in the early phases of
menstruation

e. The rapid formation of fibrin plugs and their degradation play a part of the control of
menstrual bleeding

11. Which of the following refers to WHO consensus 1999 about normosperm :

f. consentration less than 20 million/ml


g. volume at least 2 ml
h. grade A at least 50%
i. grade A + B more than 2%
j. leucocyte less than 2/ml

12.Regulation of fetal adrenal gland in relation with labor process, in late gestation :

A. The mother prevent high cortisol by converting to cortisone.

B. Increasing estogen maternal.

C. Increasing 11β- hydroxysteroid dehydrogenase.

D. A, B and C true.

E. A and C true.

13.29-year-old primigravida who received no prenatal care has marked vaginal bleeding
after the onset of labor at 38 weeks gestation. Cesarean section is performed and a
lacerated low-lying placenta is removed. She remains hypotensive for 6 hours and
requires transfusion of 12 packed RBC units. Postpartum, she becomes unable to
breast-feed the infant. She does not have a resumption of normal menstrual cycles. She
becomes more sluggish and tired. Laboratory findings include hyponatremia,
hyperkalemia, and hypoglycemia. Which of the following pathologic lesions is she most
likely to have had following delivery?
A  Bilateral adrenal hemorrhage
B   Pituitary necrosis
C   Subacute thyroiditis
D   Metastatic choriocarcinoma
E   Insulitis

14. A patient presents with amenorrhea and galactorrhea. Her prolactin level are elevated.
She is not and never has been pregnant. In addition to evaluating her for a
prolactinoma, one also needs to evaluate for other causes that would increase prolactin
such as elevated.

a. dopamine

b. gamma-aminobutyric acid

c. hystamin type II receptor activation

d. Thyrotropin releasing hormone

e. corticotrophin releasing hormone

15. The initial step in the workup of amenorheic patient after excluding pregnancy begin
with measurement of :
F. TSH
G. LH
H. AMH
I. FSH
J. GnRH

16. Which of the following sequences best describes estrogen action?

a. cell membrane diffusion, steroidreceptor,-DNA complex formation, transcription,


translation

b. cell membrane receptor activation, steroidreceotor DNA complex formation,


translation, transcription, transcription

c. cell membrane diffusion' steroidreceptor-DNA complex formation, translation,


transcription

d. cell membrane diffusion, adenylatecyclase activation, cAMP production, protein


phosphorylation

e. cell membrane receptor activation, adenylatecyclase activation, c AMP production,


protein phosphorilation..

17. Physical examination of amenorhea :


F. Body mass index
G. Breast development
H. The presence of pubic hair growth
I. Evaluation of the genital outflow and uterus
J. If all are correct

18. Which of the following causes of delayed puberty accompanies elevated circulating
gonadotropin levels?

a. Kallman's syndrome

b. Hypothalamic tumors

c. gonadaldysgenesis

d. malnutrition

e. chronic illness

19. Which of the following sequences best describes estrogen action?

a. cell membrane diffusion, steroidreceptor,-DNA complex formation, transcription,


translation

b. cell membrane receptor activation, steroidreceotor DNA complex formation,


translation, transcription, transcription

c. cell membrane diffusion' steroidreceptor-DNA complex formation, translation,


transcription

d. cell membrane diffusion, adenylatecyclase activation, cAMP production, protein


phosphorylation

e. cell membrane receptor activation, adenylatecyclase activation, c AMP production,


protein phosphorilation..

20.What happen in increase Corticotrophin Releasing Hormone (CRH) in fetal development,


All statement in bellow are true; except :

A. Increase fetal cortisol is positive feedback relationship

B. Augment fetal ACTH secretion.

C. Adjunct progesterone secretion.

D. More DHEAS.

E. Increase estrogen

You might also like